You are on page 1of 60

Total Marks : 200

Online Prelims TEST - 11 (SUBJECT WISE)


( InsightsIAS Mock Test Series for UPSC Preliminary Exam 2020 ) Mark Scored : 53.33

1 Consider the following statements regarding Balance of Payments


1. It is a systematic record of all transactions of a country with the outside world in a given year.
2. It consists of both government and private transactions.

Which of the statements given above is/are correct?


A. 1 only
B. 2 only
C. Both 1 and 2
D. Neither 1 nor 2

Your Answer : C
Correct Answer : C

Answer Justification :

Both the statements given above are correct.


0 34
6 41of a country with the
The balance of payment of a country is a systematic record of all transactions
outside world in a given year.
7 54
-8
. c om
All transactions include both private as well as government transactions with external World.

a il
2 ‘Index of eight core industries’, sometime seen in the m is released by
gnews,
2 @
m 2
A. Central Statistics Office
u pa
an
B. Department of Economic Affairs
s
d a
C. Office of Economic Adviser, Ministry of Commerce and Industry.
s
D. Department of Heavy- Industries
Da
Your Answera:m A
u p
An
Correct Answer : C

Answer Justification :

The Index is compiled and released by Office of the Economic Adviser (OEA), Department of
Industrial Policy & Promotion (DIPP), Ministry of Commerce & Industry, Government of India.

3 Consider the following statements regarding ‘most-favored-nation’ (MFN) status


1. It is one of the important principle of GATTS agreement conceived by WTO.
2. If any country grants one country a special favour such as lower customs duty same would need to
be extended to all other WTO members under MFN.

Which of the statements given above is/are correct?


A. 1 only
B. 2 only
C. Both 1 and 2
D. Neither 1 nor 2
prelims.insightsonindia.com 1
© Insights Active Learning | All rights reserved - 131815. You may not reproduce, distribute or exploit the contents in any form without
written permission by copyright owner. Copyright infringers may face civil and criminal liability
Total Marks : 200
Online Prelims TEST - 11 (SUBJECT WISE)
( InsightsIAS Mock Test Series for UPSC Preliminary Exam 2020 ) Mark Scored : 53.33

Your Answer : A
Correct Answer : B

Answer Justification :

MFN is so important a principle that it is the first article of the General Agreement on Tariffs
and Trade (GATT), which governs trade in goods. Hence Statement 1 is incorrect

Under the World Trade Organisation (WTO) agreements, countries cannot normally discriminate
between their trading partners. If any country grants one country a special favour such as a lower
customs duty rate for one of their products the same would need to be extended to all other WTO
members. This principle is known as most-favoured-nation (MFN) treatment. Hence Statement 2
is correct.

http://www.arthapedia.in/index.php?title=Most-favoured-nation_(MFN)

4
4 Which of the following is/are the features of Hybrid Annuity Model (HAM)
1 03
64
1. It is an investment model which revives PPP in highway construction in India.
4
2. In HAM, government makes first 40% payment as fixed amount in five
8 75equal installments whereas
-
the remaining 60% is paid as variable annuity amount after the completion of the project.
m
3. There is no toll right for the developer in HAM co il .
m a
@g
Which of the statements given above is/are correct?
2
m2
A. 1, 2 and 3
B. 2 only a
C. 1 only n up
a
D. 1 and 2 only as d
-
Your Answer : D as
m: A
Correct Answer
u pa
An Justification :
Answer

All the statements given above are correct.

Hybrid Annuity Model (HAM) has been introduced by the Government to revive PPP
(Public Private Partnership) in highway construction in India.

At present, three different models –PPP Annuity, PPP Toll and EPC (Engineering, Procurement and
Construction) were followed by the government while adopting private sector participation.

Here, hybrid annuity means the first 40% payment is made as fixed amount in five equal
installments whereas the remaining 60% is paid as variable annuity amount after the
completion of the project depending upon the value of assets created.

As the government pays only 40%, during the construction stage, the developer should find money
for the remaining amount.

prelims.insightsonindia.com 2
© Insights Active Learning | All rights reserved - 131815. You may not reproduce, distribute or exploit the contents in any form without
written permission by copyright owner. Copyright infringers may face civil and criminal liability
Total Marks : 200
Online Prelims TEST - 11 (SUBJECT WISE)
( InsightsIAS Mock Test Series for UPSC Preliminary Exam 2020 ) Mark Scored : 53.33

Here, he has to raise the remaining 60% in the form of equity or loans.

There is no toll right for the developer. Under HAM, Revenue collection would be the
responsibility of the National Highways Authority of India (NHAI).

http://www.arthapedia.in/index.php?title=Hybrid_Annuity_in_Infrastructure_Sector

5 India has become the fastest growing investment region for foreign investors. In this context which of
the following sectors received highest FDI in 2018-2019?

A. Service Industries
B. Computer Software and Hardware
C. Infrastructure
D. None of the above

Your Answer : B 4
Correct Answer : A 1 03
4 64
5
Answer Justification :
- 87
m
Data for 2018-19 indicates that the services sector attracted
i l . co the highest FDI equity inflow
of US$ 9.16 billion, followed by computer software a and hardware – US$ 6.42 billion, trading –
US$ 4.46 billion and telecommunications – US$ 2.67g m billion. Most recently, the total FDI equity
inflows for the month of March 2019 touched @
2US$ 3.60 billion.
m 2
u pa
n
6 Consider the following statementsaregarding FDI equity inflows in India in 2018-19
a s
- d FDI equity inflows from Mauritius
1. India received the maximum
s
Da
2. India emerged as the top recipient of greenfield FDI inflows from the Commonwealth Nations
m
astatements
u p
Which of the given above is/are correct?
A.A1nonly
B. 2 only
C. Both 1 and 2
D. Neither 1 nor 2

Your Answer : C
Correct Answer : B

Answer Justification :

During 2018-19, India received the maximum FDI equity inflows from Singapore (US$
16.23 billion), followed by Mauritius (US$ 8.08 billion), Netherlands (US$ 3.87 billion), USA (US$
3.14 billion), and Japan (US$ 2.97 billion). Hence Statement 1 is incorrect.

India emerged as the top recipient of greenfield FDI Inflows from the Commonwealth, as per a
trade review released by The Commonwealth in 2018.

prelims.insightsonindia.com 3
© Insights Active Learning | All rights reserved - 131815. You may not reproduce, distribute or exploit the contents in any form without
written permission by copyright owner. Copyright infringers may face civil and criminal liability
Total Marks : 200
Online Prelims TEST - 11 (SUBJECT WISE)
( InsightsIAS Mock Test Series for UPSC Preliminary Exam 2020 ) Mark Scored : 53.33

https://www.ibef.org/economy/foreign-direct-investment.aspx

7 Which among the following is not a sub-index in the World Bank’s “ease of doing business” index?

A. Labour Market Regulation


B. Paying taxes
C. Getting credit
D. Protecting minority investors

Your Answer : D
Correct Answer : A

Answer Justification :

The ease of doing business index is an index created by Simeon Djankov at the World Bank Group.
4 Andrei
The academic research for the report was done jointly with professors Oliver Hart3and
0
Shleifer. 41
5 46
7
-8
A nation's ranking on the index is based on the average of 10 subindices:

c
Starting a business – Procedures, time, cost, and minimum
. om capital to open a new business
il and cost to build a warehouse
Dealing with construction permits – Procedures,atime,
Getting electricity – procedures, time, and g m required for a business to obtain a permanent
cost
2 @
2
electricity connection for a newly constructed warehouse
Registering property – Procedures,
p amtime, and cost to register commercial real estate
Getting credit – Strength of u
nlegal rights index, depth of credit information index
sa on the extent of disclosure, extent of director liability, and ease
Protecting investors –aIndices
-d
of shareholder suits
s
Paying taxes D–aNumber of taxes paid, hours per year spent preparing tax returns, and total tax
payablem
p a as share of gross profit
nu across borders – Number of documents, cost, and time necessary to export and
Trading
Aimport
Enforcing contracts – Procedures, time, and cost to enforce a debt contract
Resolving insolvency – The time, cost, and recovery rate (%) under bankruptcy proceeding

https://en.wikipedia.org/wiki/Ease_of_doing_business_index

8 The Technical Education Quality Improvement Programme (TEQIP) is implemented by

A. Ministry of Skill Development and Entrepreneurship


B. Ministry of Micro, Small and Medium Enterprises
C. Ministry of Human Resource Development
D. Ministry of Labour and Employment

Your Answer : C
Correct Answer : C

prelims.insightsonindia.com 4
© Insights Active Learning | All rights reserved - 131815. You may not reproduce, distribute or exploit the contents in any form without
written permission by copyright owner. Copyright infringers may face civil and criminal liability
Total Marks : 200
Online Prelims TEST - 11 (SUBJECT WISE)
( InsightsIAS Mock Test Series for UPSC Preliminary Exam 2020 ) Mark Scored : 53.33

Answer Justification :

The Ministry of Human Resource Development has recruited more than 1200 highly qualified and
motivated graduates from IITs and NITs among others to teach in Engineering Colleges in the most
backward areas for the next 3 years.

http://www.aicte-india.org/bureaus/rifd/teqip

9 Consider the following statements


1. India's Foreign Exchange Reserves is nearly equal to10 months of India’s import.
2. Reserve Bank of India manages foreign exchange reserve in India.

Which of the statements given above is/are correct?


A. 1 only
B. 2 only
C. Both 1 and 2 4
D. Neither 1 no 2 1 03
4 64
5
Your Answer : C
- 87
Correct Answer : C
.com
Answer Justification : a i l
g m
2@
m2
Both the statements are correct.
a
India's Foreign Exchange Reserves
n up was measured at 396.0 USD bn in Aug 2019, compared
a sa
with 398.2 USD bn in the previous month. India's Foreign Exchange Reserves equaled 10.0
d
s-
Months of Import in Aug 2019
a
Reserve Bank mofDIndia manages foreign exchange reserve in India.
u pa
An
https://www.ceicdata.com/en/indicator/india/foreign-exchange-reserves

10 Which among the following measures is/are likely to result in the current account surplus in the
Balance of Payments (BoP)?
1. Fall in global crude oil prices
2. Increase in the remittances received from abroad.
3. External commercial borrowing

Select the correct answer using the code given below


A. 3 only
B. 1 and 2 only
C. 2 and 3 only
D. 1, 2 and 3

Your Answer : B
Correct Answer : B

prelims.insightsonindia.com 5
© Insights Active Learning | All rights reserved - 131815. You may not reproduce, distribute or exploit the contents in any form without
written permission by copyright owner. Copyright infringers may face civil and criminal liability
Total Marks : 200
Online Prelims TEST - 11 (SUBJECT WISE)
( InsightsIAS Mock Test Series for UPSC Preliminary Exam 2020 ) Mark Scored : 53.33

Answer Justification :

External commercial borrowing forms the component of capital account. Other measures would
result in current account surplus.

11 ‘Doha Development Agenda’, often seen in news, is

A. Latest trade-negotiation round of the World Trade Organization (WTO) to lower the trade
barriers around the world.
B. Latest climate change agreement between the western and eastern world.
C. Initiative of world community to switch renewable energy in near future
D. Development agenda to systematically address instability in Middle East.

Your Answer : A
Correct Answer : A
4
1 03
64
Answer Justification :

7
The Doha Development Round or Doha Development Agenda is the8trade-negotiation
54
round of the
-
World Trade Organization which commenced in November 2001 under then director-general Mike
m and thus facilitate increased
oworld,
.
Moore. Its objective was to lower trade barriers around thec
global trade. ail
@ gm
2 2
https://www.wto.org/english/tratop_e/dda_e/dda_e.htm
m
u pa
12 Which of the following countries a n parties of Comprehensive and Progressive Agreement for
is/are
Trans-Pacific Partnership? d a s
1. Mexico a s-
2. Japan D
3. Canada pa
m
u
An
4. China

Select the correct answer using the code given below


A. 1 and 2 only
B. 3 and 4 only
C. 1, 2, 3 and 4
D. 1, 2 and 3 only

Your Answer : A
Correct Answer : D

Answer Justification :

The Comprehensive and Progressive Agreement for Trans-Pacific Partnership (CPTPP), also known
as TPP11 or TPP-11, is a trade agreement between Australia, Brunei, Canada, Chile, Japan,
Malaysia, Mexico, New Zealand, Peru, Singapore, and Vietnam.

prelims.insightsonindia.com 6
© Insights Active Learning | All rights reserved - 131815. You may not reproduce, distribute or exploit the contents in any form without
written permission by copyright owner. Copyright infringers may face civil and criminal liability
Total Marks : 200
Online Prelims TEST - 11 (SUBJECT WISE)
( InsightsIAS Mock Test Series for UPSC Preliminary Exam 2020 ) Mark Scored : 53.33

https://en.wikipedia.org/wiki/Comprehensive_and_Progressive_Agreement_for_Trans-Pacific_Partner
ship

13 India has a Free Trade Agreement with which of the following countries/groups?
1. ASEAN
2. Sri Lanka
3. European Union

Which of the statements given above is/are correct?


A. 1 and 2 only
B. 2 only
C. 3 only
D. None

Your Answer : A
4
Correct Answer : A
1 03
4 64
Answer Justification : 5
- 87
mphase.
FTA between India and European Union is still in negotiation

i l .co
ma
https://www.india-briefing.com/news/indias-free-trade-agreements-4810.html/
g
2 2@
14 Qualified Foreign Investors are allowed tom
pa make investments in
1. T-Bills
n u
2. Corporate bonds s a
d aIndian
3. Equity and Debt schemes
s - of mutual funds
D a
p am
Which of the statements given above is/are correct?

nu
A. 3 only
B.A2 and 3 only
C. 1, 2 and 3
D. 2 only

Your Answer : B
Correct Answer : C

Answer Justification :

The Qualified Foreign Investor (QFI) is sub-category of Foreign Portfolio Investor and refers to any
foreign individuals, groups or associations, or resident, however, restricted to those from a country
that is a member of Financial Action Task Force (FATF) or a country that is a member of a group
which is a member of FATF and a country that is a signatory to International Organization of
Securities Commission’s (IOSCO) Multilateral Memorandum of Understanding (MMOU).

QFI scheme was introduced by Government of India in consultation with RBI and SEBI in the year
2011, through a Union Budget announcement.

prelims.insightsonindia.com 7
© Insights Active Learning | All rights reserved - 131815. You may not reproduce, distribute or exploit the contents in any form without
written permission by copyright owner. Copyright infringers may face civil and criminal liability
Total Marks : 200
Online Prelims TEST - 11 (SUBJECT WISE)
( InsightsIAS Mock Test Series for UPSC Preliminary Exam 2020 ) Mark Scored : 53.33

The objective of enabling QFIs is to deepen and infuse more foreign funds in the Indian capital
market and to reduce market volatility as individuals are considered to be long term investors, as
compared to institutional investors.

QFIs are allowed to make investments in the following instruments by opening a demat account in
any of the SEBI approved Qualified Depository Participant (QDP):

Equity and Debt schemes of Indian mutual funds,


Equity shares listed on recognized stock exchanges,
Equity shares offered through public offers
Corporate bonds listed/to be listed on recognized stock exchanges
G-Securities, T-Bills and Commercial Papers

http://www.arthapedia.in/index.php?title=Qualified_Foreign_Investors_(QFIs)

15 Which of the following is/are the components of current account of the Balance of Payments?
0 34
41
1. Inward Remittances
2. Dividends from Foreign Investment
4 6
3. Loans from World Bank 75
-8
Select the correct answer using the code given below
. c om
A. 1, 2 and 3 a il
B. 1 and 2 only
@ gm
C. 2 and 3 only 2 2
D. 2 only am p
u
s an
Your Answer : B a
Correct Answer : B
s -d
Da
m
Answer Justification :
u pa
Anfrom World Bank forms capital account of BOP.
Loans

16 Consider the following statements


1. Viability gap funding refers to investments made by the angel investors in budding companies.
2. Ministry of Finance stipulates FDI sector caps in various sectors of Economy

Which of the statements given above is/are correct?


A. 1 only
B. 2 only
C. Both 1 and 2
D. Neither 1 nor 2

Your Answer : B
Correct Answer : D

Answer Justification :

prelims.insightsonindia.com 8
© Insights Active Learning | All rights reserved - 131815. You may not reproduce, distribute or exploit the contents in any form without
written permission by copyright owner. Copyright infringers may face civil and criminal liability
Total Marks : 200
Online Prelims TEST - 11 (SUBJECT WISE)
( InsightsIAS Mock Test Series for UPSC Preliminary Exam 2020 ) Mark Scored : 53.33

Both the statements are incorrect.

Viability Gap Finance means a grant to support projects that are economically justified but not
financially viable.

Ministry of Commerce and Trade stipulates FDI sector caps in various sectors of Economy

17 Which of the following can reduce the deficit in Balance of Payments?


1. Reducing import custom duties
2. Restrictive monetary policy
3. Appreciation of domestic currency

Select the correct answer using the code given below


A. 3 only
B. 2 and 3 only
C. 1 and 2 only 4
D. None 1 03
4 64
5
Your Answer : D
- 87
Correct Answer : A
.com
Answer Justification : a i l
@ gm
2
Reducing import custom duties increase import 2 inflows and restrictive monetary policy reduces the
m
pa activities.
export potential due to reduced economic
u
n
a sa
Guarantee Agency. as
-d
18 Which of the following institutions is/are part of World Bank Group Multilateral Investment

1. International BankDfor Reconstruction and Development.


m
paDevelopment Association.
2. International
u
An
3. International Centre for Settlement of Investment Disputes (ICSID)

Select the correct answer using the code given below


A. 1 and 2 only
B. 1 only
C. 2 only
D. 1, 2 and 3

Your Answer : A
Correct Answer : D

Answer Justification :

All the above institutions are part of World Bank Group.

The World Bank Group (WBG) is a family of five international organizations that make leveraged
loans to developing countries. It is the largest and most well-known development bank in the world

prelims.insightsonindia.com 9
© Insights Active Learning | All rights reserved - 131815. You may not reproduce, distribute or exploit the contents in any form without
written permission by copyright owner. Copyright infringers may face civil and criminal liability
Total Marks : 200
Online Prelims TEST - 11 (SUBJECT WISE)
( InsightsIAS Mock Test Series for UPSC Preliminary Exam 2020 ) Mark Scored : 53.33

and is an observer at the United Nations Development Group

Its five organizations are the International Bank for Reconstruction and Development (IBRD), the
International Development Association (IDA), the International Finance Corporation (IFC), the
Multilateral Investment Guarantee Agency (MIGA) and the International Centre for Settlement of
Investment Disputes (ICSID). The first two are sometimes collectively referred to as the World
Bank.

19 Consider the following statements


1. International Monetary Fund can lend loan to member countries only.
2. There is a tradition of appointing a European as managing director of International Monetary Fund.

Which of the statements given above is/are correct?


A. 1 only
B. 2 only
4
03
C. Both 1 and 2
D. Neither 1 nor 2 1
4 64
5
Your Answer : C
- 87
om
Correct Answer : C

i l .c
Answer Justification :
m a
g
2@
m2
Both the statements are correct.
a
n up also known as the Fund, is an international organization
The International Monetary Fund (IMF),
a
headquartered in Washington, saD.C., consisting of 189 countries working to foster global monetary
- d stability, facilitate international trade, promote high employment and
cooperation, secure financial
s
Da growth, and reduce poverty around the world while periodically depending on
sustainable economic
amits resources.
World Bank for
p
u
An
International Monetary Fund can lend loan to member countries only.

In 2011 the world's largest developing countries, the BRIC nations, issued a statement
declaring that the tradition of appointing a European as managing director undermined
the legitimacy of the IMF and called for the appointment to be merit-based.

20 Consider the following statements regarding conditionality of loans from International Monetary
Fund
1. The IMF does not require collateral from countries for loans.
2. Nation seeking assistance from IMF has to correct its macroeconomic imbalances in the form of
policy reform.

Which of the statements given above is/are correct?


A. 1 only
B. 2 only
C. Both 1 and 2

prelims.insightsonindia.com 10
© Insights Active Learning | All rights reserved - 131815. You may not reproduce, distribute or exploit the contents in any form without
written permission by copyright owner. Copyright infringers may face civil and criminal liability
Total Marks : 200
Online Prelims TEST - 11 (SUBJECT WISE)
( InsightsIAS Mock Test Series for UPSC Preliminary Exam 2020 ) Mark Scored : 53.33

D. Neither 1 nor 2

Your Answer : C
Correct Answer : B

Answer Justification :

Conditionality of loans

IMF conditionality is a set of policies or conditions that the IMF requires in exchange for financial
resources.

The IMF does require collateral from countries for loans but also requires the government
seeking assistance to correct its macroeconomic imbalances in the form of policy reform.
Hence Statement is incorrect.

4 introduced
If the conditions are not met, the funds are withheld. The concept of conditionality was
3
in a 1952 Executive Board decision and later incorporated into the Articles of 1 0
Agreement. Hence
6 4
Statement 2 is correct. 4 5
7
-8
omforeign exchange reserves?
21 Which one of the following groups of items is included in India’s
.c
a i l
A. gmRBI and loans from the World Bank.
Foreign currency assets, gold holdings of the
@
B. 2 Rights (SDRs) and loans from foreign countries.
Foreign-currency assets, Special Drawing
2
C.
am World Bank and SDRs.
Foreign-currency assets, loans from
p
the
D. u holdings of the RBI and SDRs.
Foreign-currency assets, gold
s an
Your Answer : D - da
Correct Answer : D as
D
Answer u p am
Justification :
An
Reserve Bank of India Act and the Foreign Exchange Management Act, 1999 set the legal
provisions for governing the foreign exchange reserves. Reserve Bank of India accumulates foreign
currency reserves by purchasing from authorized dealers in open market operations. Foreign
exchange reserves of India act as a cushion against rupee volatility once global interest rates start
rising.

The Foreign exchange reserves of India consists of below four categories;[9]

Foreign Currency Assets


Gold
Special Drawing Rights (SDRs)
Reserve Tranche Position.

22 Consider the following statements


1. Foreigner visit to Indian tourist place can be considered as export from India.
2. Indian medical professional offering services through telemedicine forms the component of import.

prelims.insightsonindia.com 11
© Insights Active Learning | All rights reserved - 131815. You may not reproduce, distribute or exploit the contents in any form without
written permission by copyright owner. Copyright infringers may face civil and criminal liability
Total Marks : 200
Online Prelims TEST - 11 (SUBJECT WISE)
( InsightsIAS Mock Test Series for UPSC Preliminary Exam 2020 ) Mark Scored : 53.33

Which of the statements given above is/are correct?


A. 1 only
B. 2 only
C. Both 1 and 2
D. Neither 1 nor 2

Your Answer : A
Correct Answer : A

Answer Justification :

Foreigner visit to Indian tourist place can be considered as export from India. Hence Statement 1
is correct.

Indian medical professional offering services through telemedicine forms the component of export.
Hence Statement 2 is incorrect
4
1 03
23 With reference to Agreement on Agriculture (AoA), Peace clause refers to 4 64
7 5
-8
A. Mechanism to supply food grains to least developed countries at nominal rates
m for avoiding distortion in trade
onations
B. Peace agreement between developing and developed
il . c
C. Temporary arrangement available to developinganations, including India, till a permanent
solution is found to public stockholding forg
m
food security purposes.
@
22are correct.
D. None of the statements (a), (b), and (c)
m
u pa
Your Answer :
a n
s
Correct Answer : C
da
a s-
D :
Answer Justification
m
u pa have taken steps to reform the agriculture sector and to address the subsidies and
WTO members
highA
n
trade barriers that distort agricultural trade. The overall aim is to establish a fairer trading
system that will increase market access and improve the livelihoods of farmers around the world.
The WTO Agreement on Agriculture, which came into force in 1995, represents a significant step
towards reforming agricultural trade and making it fairer and more competitive. The Agriculture
Committee oversees implementation of the Agreement.

Members continue to conduct negotiations for further reform. In 2015, they adopted a historic
decision to abolish agricultural export subsidies and to set rules for other forms of farm export
support.

The Agreement in Agriculture contains a “due restraint” or “peace clause” which


regulates the application of other WTO agreements to subsidies in respect of agricultural
products (Article 13). The provisions provide that Green Box domestic support measures cannot
be the subject of countervailing duty action or other subsidy action under the WTO Agreement on
Subsidies and Countervailing Measures, nor can they be subject to actions based on non-violation
nullification or impairment of tariff concessions under the GATT.

prelims.insightsonindia.com 12
© Insights Active Learning | All rights reserved - 131815. You may not reproduce, distribute or exploit the contents in any form without
written permission by copyright owner. Copyright infringers may face civil and criminal liability
Total Marks : 200
Online Prelims TEST - 11 (SUBJECT WISE)
( InsightsIAS Mock Test Series for UPSC Preliminary Exam 2020 ) Mark Scored : 53.33

https://www.wto.org/english/tratop_e/agric_e/agric_e.htm

https://www.wto.org/english/tratop_e/agric_e/ag_intro05_other_e.htm

24 With reference to the Currency Market, the term “Convertibility” is used to denote
1. Freedom to exchange currencies like commodities across the countries.
2. Freedom to invest in different countries
3. Freedom to residents to remit outside the country.

Which of the statements given above is/are correct?


A. 1 only
B. 2 and 3 only
C. 3 only
D. None

4
03
Your Answer : A
Correct Answer : A 1
4 64
5
Answer Justification :
- 87
Currency convertibility is the ease with which a country's.c o m
currency can be converted into gold or
another currency. Currency convertibility is important a i l
for international commerce as globally
sourced goods must be paid for in an agreed upon
m
g currency that may not be the buyer's domestic
2 @
m2
currency.
a
n up
25 Consider the following statements
a sa regarding New Development Bank (NDB)
- d bank established by BRICS.
1. It is a multilateral development
s
2. Bank supports public
D a or private projects through loans, guarantees and equity participation.
am office of the NDB is in New Delhi.
3. The first regional
p
u
Which Anof the statements given above is/are correct?
A. 1 and 2 only
B. 2 only
C. 1 and 3 only
D. 1, 2 and 3

Your Answer : D
Correct Answer : A

Answer Justification :

The New Development Bank (NDB), formerly referred to as the BRICS Development Bank, is a
multilateral development bank established by the BRICS states (Brazil, Russia, India, China and
South Africa). Hence Statement 1 is correct.

According to the Agreement on the NDB, "the Bank shall support public or private projects through
loans, guarantees, equity participation and other financial instruments." Moreover, the NDB "shall

prelims.insightsonindia.com 13
© Insights Active Learning | All rights reserved - 131815. You may not reproduce, distribute or exploit the contents in any form without
written permission by copyright owner. Copyright infringers may face civil and criminal liability
Total Marks : 200
Online Prelims TEST - 11 (SUBJECT WISE)
( InsightsIAS Mock Test Series for UPSC Preliminary Exam 2020 ) Mark Scored : 53.33

cooperate with international organizations and other financial entities, and provide technical
assistance for projects to be supported by the Bank. Hence Statement 2 is correct.

The bank is headquartered in Shanghai, China.

The first regional office of the NDB is in Johannesburg, South Africa. Hence Statement 3 is
incorrect.

26 Consider the following statements regarding Intellectual Property Rights policy 2016
1. It is entirely in compliant with the WTO’s agreement on TRIPS
2. It aims to remove Compulsory Licensing clause from India's Patents Act.

Which of the statements given above is/are correct?


A. 1 only
B. 2 only
4
03
C. Both 1 and 2
D. Neither 1 nor 2 1
4 64
5
Your Answer : C
- 87
Correct Answer : A
.com
Answer Justification : a i l
@ gm
2 compliance with WTO's (World Trade Organisation)
Intellectual Property Rights Policy which is2in
am of IPRs), aims to sustain entrepreneurship. Hence
agreement on TRIPS (Trade Related aspects
p
Statement 1 is correct. n u
a sa
s -d
It suggests making the department of industrial policy and promotion (DIPP) the nodal agency for
all IPR issues.
D a
m
a norms, a CL can be invoked by a government allowing a company to produce a
As per thepWTO
u
An product without the consent of the patent owner in public interest. Under the Indian
patented
Patents Act, a CL can be issued for a drug if the medicine is deemed unaffordable, among other
conditions, and the government grants permission to qualified generic drug makers to manufacture
it. Hence Statement 2 is incorrect.

https://www.thehindu.com/business/all-you-need-to-know-about-the-intellectual-property-rights-polic
y/article8600530.ece

27 ‘SWISS Challenge Model’, often in news, is

A. International air traffic management model implemented in UDAAN mission.


B. Operation to uncover black money from Switzerland.
C. Challenge to clean corrupt practices in taxation system in India
D. One of the method of awarding government contracts to private players.

Your Answer : D

prelims.insightsonindia.com 14
© Insights Active Learning | All rights reserved - 131815. You may not reproduce, distribute or exploit the contents in any form without
written permission by copyright owner. Copyright infringers may face civil and criminal liability
Total Marks : 200
Online Prelims TEST - 11 (SUBJECT WISE)
( InsightsIAS Mock Test Series for UPSC Preliminary Exam 2020 ) Mark Scored : 53.33

Correct Answer : D

Answer Justification :

A Swiss Challenge is a method of bidding, often used in public projects, in which an interested party
initiates a proposal for a contract or the bid for a project. The government then puts the details of
the project out in the public and invites proposals from others interested in executing it. On the
receipt of these bids, the original contractor gets an opportunity to match the best bid.

https://www.thehindubusinessline.com/opinion/columns/slate/all-you-wanted-to-know-about-swiss-ch
allenge/article24194034.ece

28 The price of any currency in international market can be fluctuated by which of the following factors?
1. Demand for goods/services provided by the country concerned.
2. Stability of the government of the concerned country.
3. World Bank 4 3
10
Select the correct answer using the code given below 4 64
5
A. 1 only
- 87
B. 1 and 2 only
C. 2 and 3 only . c om
D. None ail m
g
Your Answer : B 2 2@
Correct Answer : B am
up
s an
Answer Justification :
da
s-
Daplays no role in fluctuation of any country’s currency.
World Bank nearly

p am
u
Anthe following statements
29 Consider
1. Currency devaluation can help to reduce current account deficit.
2. Full capital account convertibility brings stability to an economy.

Which of the statements given above is/are correct?


A. 1 only
B. 2 only
C. Both 1 and 2
D. Neither 1 nor 2

Your Answer : A
Correct Answer : A

Answer Justification :

Through Full Capital account convertibility a company or person can withdraw long term
investments from a country at any point of time. Thus, bringing economic instability. Hence

prelims.insightsonindia.com 15
© Insights Active Learning | All rights reserved - 131815. You may not reproduce, distribute or exploit the contents in any form without
written permission by copyright owner. Copyright infringers may face civil and criminal liability
Total Marks : 200
Online Prelims TEST - 11 (SUBJECT WISE)
( InsightsIAS Mock Test Series for UPSC Preliminary Exam 2020 ) Mark Scored : 53.33

Statement 2 is incorrect.

30 Consider the following statements regarding ‘World Intellectual Property Organization’ (WIPO):
1. It is one of the specialized agency of European Union.
2. It was created to promote and encourage creative activity in the world.

Which of the statements given above is/are correct?


A. 1 only
B. 2 only
C. Both 1 and 2
D. Neither 1 nor 2

Your Answer : B
Correct Answer : B

4
Answer Justification :
1 03
64
The World Intellectual Property Organization is one of the 15 specialized4agencies] of the United
5
Nations (UN). Hence Statement 1 is incorrect. 87-
m
WIPO was created in 1967 "to encourage creative activity,
i l .ctoo promote the protection of intellectual
property throughout the world". Hence Statement 2ais correct.
@ gm
2 2
https://en.wikipedia.org/wiki/World_Intellectual_Property_Organization
m
u pa
an of domestic economy and aids in increasing foreign exchange
31 Foreign trade facilitates the growth
s
d a
reserve. With reference to foreign trade, consider the following statements:
-
asexports is around 10% in 2017-2018 fiscal year.
1. India's share in world
D
2. Crude oil and its processed products are the principal commodity of both import and exports
p
respectively ainmIndia.
u
An
Which of the statements given above is/are correct?
A. 1 only
B. 2 only
C. Both 1 and 2
D. Neither 1 nor 2

Your Answer : C
Correct Answer : B

Answer Justification :

India’s share in world exports is around 1.6%. Hence Statement 1 is incorrect.

prelims.insightsonindia.com 16
© Insights Active Learning | All rights reserved - 131815. You may not reproduce, distribute or exploit the contents in any form without
written permission by copyright owner. Copyright infringers may face civil and criminal liability
Total Marks : 200
Online Prelims TEST - 11 (SUBJECT WISE)
( InsightsIAS Mock Test Series for UPSC Preliminary Exam 2020 ) Mark Scored : 53.33

4
1 03
4 64
5
- 87
.com
a i l
g m
2@
m2 List of the Seventh Schedule in constitution.
32 Consider the following statements
a
uproute, is not allowed in food processing Industries.
1. Contract Farming defined under Concurrent
n
a
2. FDI up-to 100% under the automatic
s
a
-d
s given above is/are correct?
Which of the statements
D a
A. 1 only
m
pa
B. 2 only
u
An 1 and 2
C. Both
D. Neither 1 nor 2

Your Answer : A
Correct Answer : A

Answer Justification :

Contract Farming can be carried out based on a pre-harvest agreement between


buyers and producers. It is defined under Concurrent List of the Seventh Schedule of
constitution; however Agriculture is under State list. Hence statement 1 is correct.

FDI up to 100%, under the automatic route, is allowed in food processing industries. Further,
100% FDI under Government route for retail trading, including through e-commerce, is permitted in
respect of food products manufactured and/or produced in India. Hence statement 2 is incorrect.

33 Consider the following statements

prelims.insightsonindia.com 17
© Insights Active Learning | All rights reserved - 131815. You may not reproduce, distribute or exploit the contents in any form without
written permission by copyright owner. Copyright infringers may face civil and criminal liability
Total Marks : 200
Online Prelims TEST - 11 (SUBJECT WISE)
( InsightsIAS Mock Test Series for UPSC Preliminary Exam 2020 ) Mark Scored : 53.33

1. Ministry of Health & Family Welfare administrates Food Safety and Standards Act, 2006
2. National Programme for Organic Production (NPOP) is implemented by Ministry of Agriculture and
Farmer’s welfare.

Which of the statements given above is/are correct?


A. 1 only
B. 2 only
C. Both 1 and 2
D. Neither 1 nor 2

Your Answer : C
Correct Answer : A

Answer Justification :

Food Safety and Standards Act, 2006 is an integrated food law that lays down standards and
34food sector. It
guidelines for consumer safety, protection of consumer health and regulation of0the
seeks to harmonise Indian standards with the international standards
6 41like CODEX and
facilitates international trade in food articles. 5 4
7
-8
It is a comprehensive legislation for the sector and subsumes
o m the then existing acts and
c
il.of 1954 ,Fruit Products Order of 1955,
standards like Prevention of Food Adulteration Act(PFA)
a
gm
Meat Food Products Order of 1973, Vegetable Oil Products (Control) Order of 1947, Edible Oils
Packaging (Regulation)Order of 1988, Solvent@ Extracted Oil, De- Oiled Meal and Edible Flour
2 2 Order of 1992 and also any order issued under the
(Control) Order of 1967, Milk and Milk Products
m
pa to food .
Essential Commodities Act, 1955 relating
u
n
sa
da Welfare administrates Food Safety and Standards Act, 2006.
Ministry of Health & Family
-
Hence Statement 1 sis correct.
Da
m
Ministry ofaCommerce has implemented the National Programme for Organic Production
u p
n since 2001. The objectives of NPOP are as under:
(NPOP)
A
1) To provide the means of evaluation of certification programme for organic agriculture and
products (including wild harvest, aquaculture, livestock products) as per the approved criteria.

2) To accredit certification programmes of Certification Bodies seeking accreditation.

3) To facilitate certification of organic products in conformity with the prescribed standards.

4) To facilitate certification of organic products in conformity with the importing countries organic
standards as per equivalence agreement between the two countries or as per importing country
requirements.

5) To encourage the development of organic farming and organic processing. Hence Statement 2
is incorrect.

34 Consider the following statements regarding National Policy on Biofuels-2018

prelims.insightsonindia.com 18
© Insights Active Learning | All rights reserved - 131815. You may not reproduce, distribute or exploit the contents in any form without
written permission by copyright owner. Copyright infringers may face civil and criminal liability
Total Marks : 200
Online Prelims TEST - 11 (SUBJECT WISE)
( InsightsIAS Mock Test Series for UPSC Preliminary Exam 2020 ) Mark Scored : 53.33

1. Policy indicates a viability gap funding scheme for 2G ethanol Bio refineries of Rs.5000 crore in 6
years.
2. Policy assures minimum income support for farmers for the production of bio-fuels
3. Roles and responsibilities of all the concerned Ministries/Departments with respect to biofuels has
been captured in the Policy document to synergise efforts.

Which of the statements given above is/are correct?


A. 2 only
B. 1 and 3 only
C. 3 only
D. 1, 2 and 3

Your Answer : D
Correct Answer : B

Answer Justification :
4
1 03
National Policy on Biofuels – 2018.
4 64
5
Salient Features:
- 87
.c om
i l
ma
1. The Policy categorises biofuels as "Basic Biofuels" viz. First Generation (1G) bioethanol &
biodiesel and "Advanced Biofuels" - SecondgGeneration (2G) ethanol, Municipal Solid Waste
2 2@(3G) biofuels, bio-CNG etc. to enable extension of
(MSW) to drop-in fuels, Third Generation
m
u pa
appropriate financial and fiscal incentives under each category.
n
a sa
2. The Policy expands
s -dthe scope of raw material for ethanol production by allowing use of
a Sugar containing materials like Sugar Beet, Sweet Sorghum, Starch
Sugarcane Juice,
D
p amunfit
containing materials like Corn, Cassava, Damaged food grains like wheat, broken rice, Rotten

n u
Potatoes, for human consumption for ethanol production.
A
3. Farmers are at a risk of not getting appropriate price for their produce during the surplus
production phase. Taking this into account, the Policy allows use of surplus food grains for
production of ethanol for blending with petrol with the approval of National Biofuel
Coordination Committee.

4. With a thrust on Advanced Biofuels, the Policy indicates a viability gap funding
scheme for 2G ethanol Bio refineries of Rs.5000 crore in 6 years in addition to
additional tax incentives, higher purchase price as compared to 1G biofuels.

5. The Policy encourages setting up of supply chain mechanisms for biodiesel production from
non-edible oilseeds, Used Cooking Oil, short gestation crops.

6. Roles and responsibilities of all the concerned Ministries/Departments with respect

prelims.insightsonindia.com 19
© Insights Active Learning | All rights reserved - 131815. You may not reproduce, distribute or exploit the contents in any form without
written permission by copyright owner. Copyright infringers may face civil and criminal liability
Total Marks : 200
Online Prelims TEST - 11 (SUBJECT WISE)
( InsightsIAS Mock Test Series for UPSC Preliminary Exam 2020 ) Mark Scored : 53.33

to biofuels has been captured in the Policy document to synergise efforts.

35 Consider the following statements regarding Khadi and Village Industry Commission (KVIC)
1. KVIC is a statutory body under the Ministry of Agriculture and Farmer’s Welfare
2. It promotes research in the production techniques and provide financial assistance for the Khadi
and Village Industries sector.
3. It implements Prime Minister’s Employment Generation Programme

Which of the statements given above is/are correct?


A. 2 and 3 only
B. 1 and 2 only
C. 2 only
D. 1, 2 and 3

Your Answer : A
4
Correct Answer : A
1 03
4 64
Answer Justification : 5
- 87
The Khadi and Village Industries Commission (KVIC) is m a statutory body established by an Act
. c o
a i l
of Parliament. In April 1957, it took over the work of former All India Khadi and Village Industries
Board.
@ gm
KVIC works under the overall guidance of 2 2
Ministry of Micro, Small and Medium Enterprises.
Hence Statement 1 is incorrect. am
n up
sa planning, promotion, organisation and implementation of
The KVIC is charged withathe
- d of Khadi and other village industries in the rural areas in
programs for the development
s
coordination withD a agencies engaged in rural development wherever necessary. Hence
other
Statement 2 m
p a is correct.
u
Animplements Prime Minister’s Employment Generation Programme. Hence Statement 3 is
It also
correct.

36 Voluntary increase in the value of domestic currency against a foreign currency is best described as

A. Deprecation
B. Revaluation
C. Devaluation
D. Appreciation

Your Answer : B
Correct Answer : B

Answer Justification :

A revaluation is a calculated upward adjustment to a country's official exchange rate relative to a

prelims.insightsonindia.com 20
© Insights Active Learning | All rights reserved - 131815. You may not reproduce, distribute or exploit the contents in any form without
written permission by copyright owner. Copyright infringers may face civil and criminal liability
Total Marks : 200
Online Prelims TEST - 11 (SUBJECT WISE)
( InsightsIAS Mock Test Series for UPSC Preliminary Exam 2020 ) Mark Scored : 53.33

chosen baseline. The baseline can include wage rates, the price of gold, or a foreign currency.

Revaluation is the opposite of devaluation, which is a downward adjustment.

37 Consider the following statements


1. Fair Price Shops are licensed under National Food Security Act, 2013 to distribute food grains
under Targeted Public Distribution Scheme.
2. There is no official definition for Ration card in India

Which of the statements given above is/are correct?


A. 1 only
B. 2 only
C. Both 1 and 2
D. Neither 1 nor 2

4
03
Your Answer : C
1
64
Correct Answer : D
5 4
Answer Justification :
- 87
Both the statements given above are incorrect. .com
a i l
gm to distribute essential commodities by
Fair Price Shop means a shop which has been licensed
@
an order issued under section 3 of the 2 2
Essential Commodities Act, 1955, to the ration card
am
holders under the Targeted Public Distribution System.
n up
sa 2(4) of National Food Security Act, 2013.
The term is defined in Section
a
s -d
Ration card meansaa document issued under an order or authority of the State Government for the
D commodities from the fair price shops under the Public Distribution System
m
purchase of essential
pa Public Distribution System (TPDS).
(PDS) / Targeted
u
An
Ration Card is issued per family. It is voluntary and not compulsory for citizens to acquire it.
However, all those citizens who want to get subsidised food may need to get that.

Though ration cards have been in use for a very long time, the term got legally defined
later in Section 2(16) of National Food Security Act, 2013.

38 Consider the following statements regarding Codex Alimentarius Commission (CAC)


1. It is an intergovernmental body with over 180 members.
2. It was established by the Food and Agriculture Organization of the United Nations (FAO) and the
World Health Organization (WHO) in 1960’s

Which of the statements given above is/are correct?


A. 1 only
B. 2 only
C. Both 1 and 2

prelims.insightsonindia.com 21
© Insights Active Learning | All rights reserved - 131815. You may not reproduce, distribute or exploit the contents in any form without
written permission by copyright owner. Copyright infringers may face civil and criminal liability
Total Marks : 200
Online Prelims TEST - 11 (SUBJECT WISE)
( InsightsIAS Mock Test Series for UPSC Preliminary Exam 2020 ) Mark Scored : 53.33

D. Neither 1 nor 2

Your Answer :
Correct Answer : C

Answer Justification :

Both the statements are correct.

The Codex Alimentarius Commission (CAC) is an intergovernmental body with over 180
members, within the framework of the Joint Food Standards Programme established by the Food
and Agriculture Organization of the United Nations (FAO) and the World Health Organization
(WHO), with the purpose of protecting the health of consumers and ensuring fair practices in the
food trade.

It was established in 1960’s.

The Codex Alimentarius is recognized by the World Trade Organization1as 0 an international 34


6 4
reference point for the resolution of disputes concerning food safety and4consumer protection.
7 5
-8
39 Consider the following statements:
. c om
a l
1. World Food Programme is the food assistance programiadministered by International Food Policy
Research Institute.
@ gm
2 by United Nations Forum on Forest.
2. State of the World’s Forests report is published
a m2
upis/are correct?
Which of the statements given above
n
A. 1 only
a sa
B. 2 only
s -d
C. Both 1 and 2 a
D. Neitherm
D
1 nor 2
u pa
n
YourAAnswer :
Correct Answer : D

Answer Justification :

Both the statements are incorrect.

The World Food Programme (WFP) is the food-assistance branch of the United Nations and the
world's largest humanitarian organization addressing hunger and promoting food security.

It is a member of the United Nations Development Group and part of its executive committee.
It has headquarters in Rome and from more than 80 country offices around the world.

WFP works to help people who cannot produce or obtain enough food for themselves and their
families.

The State of the World's Forests published by Food and Agriculture organization and it

prelims.insightsonindia.com 22
© Insights Active Learning | All rights reserved - 131815. You may not reproduce, distribute or exploit the contents in any form without
written permission by copyright owner. Copyright infringers may face civil and criminal liability
Total Marks : 200
Online Prelims TEST - 11 (SUBJECT WISE)
( InsightsIAS Mock Test Series for UPSC Preliminary Exam 2020 ) Mark Scored : 53.33

reports on the status of forests, recent major policy and institutional developments and key issues
concerning the forest sector.

40 Consider the following statements regarding Indian Fisheries


1. West Bengal is the leading producer of fisheries followed by Andhra Pradesh
2. About 75% of the marine fish landings are in the west coast and around 25% is contributed by east
coast.

Which of the statements given above is/are correct?


A. 1 only
B. 2 only
C. Both 1 and 2
D. Neither 1 nor 2

Your Answer : C
4
Correct Answer : B
1 03
4 64
Answer Justification : 5
- 87
Andhra Pradesh is the leading producer of fisheries. Hencem
Statement 1 is incorrect.
.co
a i l
g m
2
Distribution of Fish Production in India 2@
m
u pa
a n
s
1. AP - da
s
2. WB Da
3. Gujarat am
4. Keralaup
An Nadu
5. Tamil
6. Maharahtra

It is estimated that about 75% of the marine fish landings are in the west coast and only 25% is
contributed by east coast. Hence Statement 2 is correct.

41 Which of the following are sub- schemes under Rashtriya Krishi Vikas YojanaRemunerative
Approaches for Agriculture and Allied sector Rejuvenation (RKVYRAFTAAR)
1. Crop Diversification Program (CDP)
2. Saffron Mission
3. Bringing Green Revolution to Eastern India (BGREI)

Which of the statements given above is/are correct?


A. 1 and 3 only
B. 1, 2 and 3
C. 3 only

prelims.insightsonindia.com 23
© Insights Active Learning | All rights reserved - 131815. You may not reproduce, distribute or exploit the contents in any form without
written permission by copyright owner. Copyright infringers may face civil and criminal liability
Total Marks : 200
Online Prelims TEST - 11 (SUBJECT WISE)
( InsightsIAS Mock Test Series for UPSC Preliminary Exam 2020 ) Mark Scored : 53.33

D. None

Your Answer :
Correct Answer : B

Answer Justification :

The Cabinet Committee on Economic Affairs chaired by the Prime Minister


Shri Narendra Modi, has approved the continuation of Rashtriya Krishi Vikas
Yojana (RKVY) as Rashtriya Krishi Vikas Yojana- Remunerative Approaches
for Agriculture and Allied sector Rejuvenation (RKVY-RAFTAAR) for three
years i.e. 2017-18 to 2019-20.
The financial allocation of the scheme will be Rs. 15,722 crore with the objective
of making farming as a remunerative economic activity through strengthening
the farmer's effort, risk mitigation and promoting agribusiness
entrepreneurship.
It will incentivize states to increase allocations for agriculture and allied sectors 34
0
and help in creation of post-harvest infrastructure and promotion of private 41
6
investment in the farm sector across the country 54 7
-8
om
RKVY-RAFTAAR sub-schemes
. c
ilof Problem Soil (RPS).
Bringing Green Revolution to Eastern India (BGREI).
a
gm(FMDCP).
Crop Diversification Program (CDP). Reclamation
Foot & Mouth Disease – Control Program
2 @
Saffron Mission.
m 2
pa Programme (AFDP)
Accelerated Fodder Development
a nu
as
http://vikaspedia.in/agriculture/policies-and-schemes/cropsrelated/rashtriya-krishi-vikas-yojana-1/ra
d
s-
shtriya-krishi-vikas-yojana
a
D
a m
u
42 Consider the pfollowing statements
A n
1. Co-operatives are the largest institutional lenders for agriculture sector in India.
2. Kisan Vikas Patra is a tax saving instrument where individuals can invest money for betterment of
agriculture sector in India

Which of the statements given above is/are correct?


A. 1 only
B. 2 only
C. Both 1 and 2
D. Neither 1 nor 2

Your Answer : C
Correct Answer : D

Answer Justification :

Both the statements are incorrect.

prelims.insightsonindia.com 24
© Insights Active Learning | All rights reserved - 131815. You may not reproduce, distribute or exploit the contents in any form without
written permission by copyright owner. Copyright infringers may face civil and criminal liability
Total Marks : 200
Online Prelims TEST - 11 (SUBJECT WISE)
( InsightsIAS Mock Test Series for UPSC Preliminary Exam 2020 ) Mark Scored : 53.33

Commercial banks are the largest institutional lenders for agriculture sector in India.

Kisan Vikas Patra (KVP) is a saving instrument (But not tax saving instrument) launched by the
Government for individual savers, wherein invested money doubled during the maturity period.

This savings scheme was first launched by the Government on 1 April, 1988 and was distributed
through post offices. It was discontinued in 2011 and later reintroduced in 2014.

The maturity period of the scheme is around 100 months and the money invested doubled on
maturity. However, KVP is not a tax saving instrument as it does not offer any income tax
exemption.

43 Consider the following statements


1. Long Term Irrigation Fund had been constituted to rejuvenate local ponds and effective
management of local watershed system.

3 4
2. Agriculture census has been conducting in India from 1950’s and last census was conducted in
2015. 10 4
6
Which of the statements given above is/are correct? 7 54
A. 1 only -8
B. 2 only
. c om
C. Both 1 and 2 a il
D. Neither 1 nor 2 gm @
2 2
m
Your Answer : C
u pa
Correct Answer : D
a n
s
Answer Justification :-
da
D as
a m
Long Term Irrigation Fund (LTIF) was created in 2016 with an initial corpus of Rs. 20,000 crore
p
u and fast tracking the implementation of incomplete major and medium
for funding
An projects.
irrigation

It aims to bridge the resource gap and facilitate completion of these projects during 2016-2020.
LTIF has been instituted in NABARD under PMKSY (Pradhan Mantri Krishi Sinchayee Yojana)

Agricultural Census, which is conducted every five years in India. It is the largest countrywide
statistical operation undertaken by Ministry of Agriculture, for collection of data on structure of
operational holdings by different size classes and social groups.

Agricultural Census is carried out as a Central Sector Scheme under which 100% financial
assistance is provided to States/Union Territoriess. Agricultural Census operation is carried out in
three phases.

The first Agricultural Census in the country was conducted with reference
year 1970-71.

http://www.arthapedia.in/index.php?title=Agricultural_Census

prelims.insightsonindia.com 25
© Insights Active Learning | All rights reserved - 131815. You may not reproduce, distribute or exploit the contents in any form without
written permission by copyright owner. Copyright infringers may face civil and criminal liability
Total Marks : 200
Online Prelims TEST - 11 (SUBJECT WISE)
( InsightsIAS Mock Test Series for UPSC Preliminary Exam 2020 ) Mark Scored : 53.33

44 Which one of the following statements regarding India's Strategic Crude Oil Reserve Programme is
not correct?

A. The reserves will be developed along the coastal areas.


B. The crude oil storages are kept in underground rock caverns instead of conventional tanks.
C. It aims to take advantage of the slump in global crude oil prices.
D. All the statements given above are correct.

Your Answer : A
Correct Answer : D

Answer Justification :

45 Consider the following statements regarding Special Drawing Rights (SDR)


1. The SDR is an international reserve asset, created by the IMF in 1969 to supplement its member
countries’ official reserves.
2. The SDR is neither a currency nor a claim on the IMF. 0 34
6 41 countries’ official
3. SDR allocations can play a role in providing liquidity and supplementing member
4
reserves. 75
-8
Which of the statements given above is/are correct?
. c om
A. 2 only a il
B. 1 only
@ gm
C. 1 and 3 only
2 2
D. 1, 2 and 3 am p
u
Your Answer : D s an
a
Correct Answer : D -d
D as
p am
Answer Justification :
u
Anstatements given above are correct.
All the

The SDR is an international reserve asset, created by the IMF in 1969 to supplement its
member countries’ official reserves. So far SDR 204.2 billion (equivalent to about US$291
billion) have been allocated to members, including SDR 182.6 billion allocated in 2009 in the wake
of the global financial crisis. The value of the SDR is based on a basket of five currencies—the U.S.
dollar, the euro, the Chinese renminbi, the Japanese yen, and the British pound sterling.

The SDR serves as the unit of account of the IMF and some other international organizations. The
SDR is neither a currency nor a claim on the IMF. Rather, it is a potential claim on the freely
usable currencies of IMF members. SDRs can be exchanged for these currencies.

The collapse of Bretton Woods system in 1973 and the shift of major currencies to floating exchange
rate regimes lessened the reliance on the SDR as a global reserve asset. Nonetheless, SDR
allocations can play a role in providing liquidity and supplementing member countries’
official reserves, as was the case with the 2009 allocations totaling SDR 182.6 billion to IMF
members amid the global financial crisis.

prelims.insightsonindia.com 26
© Insights Active Learning | All rights reserved - 131815. You may not reproduce, distribute or exploit the contents in any form without
written permission by copyright owner. Copyright infringers may face civil and criminal liability
Total Marks : 200
Online Prelims TEST - 11 (SUBJECT WISE)
( InsightsIAS Mock Test Series for UPSC Preliminary Exam 2020 ) Mark Scored : 53.33

46 Consider the following statements regarding Invest India


1. It is national investment promotion and facilitation agency of India.
2. It act as the first point of reference for investors in India.
3. It is a non-profit venture under the Ministry of Finance

Which of the statements given above is/are correct?


A. 2 and 3 only
B. 1 and 3 only
C. 1, 2 and 3
D. 1 and 2 only

Your Answer :
Correct Answer : C

Answer Justification :
4
All the statements given above are correct.
1 03
64
As the national investment promotion and facilitation agency, Invest
7 54 India focuses on sector-
specific investor targeting and development of new partnerships -to8enable sustainable investments
in India. om .c
a i l
m in India.
It act as the first point of reference for investors
2 @g
a m2and multilateral
In addition to a core team that focuses on sustainable investments, Invest India also partners with

u p
substantial investment promotion agencies organizations. Invest India also actively
works with several Indian states
s anto build capacity as well as bring in global best practices in
d a
investment targeting, promotion and facilitation areas.
-
Invest India, setD asin 2009, is a non-profit venture under the Department for Promotion
up
m Internal Trade, Ministry of Commerce and Industry, Government of India.
of Industryaand
u p
A n
https://www.investindia.gov.in/about-us

47 Consider the following statements regarding India’s foreign Trade


1. Exports has seen continuous increase from last 10 years
2. Exports of services forms the important component of export from India.

Which of the statements given above is/are correct?


A. 1 only
B. 2 only
C. Both 1 and 2
D. Neither 1 nor 2

Your Answer : B
Correct Answer : B

Answer Justification :

prelims.insightsonindia.com 27
© Insights Active Learning | All rights reserved - 131815. You may not reproduce, distribute or exploit the contents in any form without
written permission by copyright owner. Copyright infringers may face civil and criminal liability
Total Marks : 200
Online Prelims TEST - 11 (SUBJECT WISE)
( InsightsIAS Mock Test Series for UPSC Preliminary Exam 2020 ) Mark Scored : 53.33

Exports has seen slump in 2014 and 2015 fiscal year.

Service exports has become one of the mainstay of India's total exports increasing manifold, from
Rs. 0.746 lakh crore in 2000-01 to Rs. 14.389 lakh crore in 2018-19, raising its share in total exports
from 26.8 per cent to 38.4 per cent.

https://pib.gov.in/newsite/PrintRelease.aspx?relid=191212

4
1 03
4 64
5
- 87
.com
a i l
g m
2 2@
am
up
s an
- da
s
Da
m
u pa
An

48 Consider the following statements


1. From last two fiscal years, food subsidies provided by the central government is greater than
fertilizer subsidies
2. According to Interim Budget 2019-2020, interest payment would be a major expenditure of
government

Which of the statements given above is/are correct?


A. 1 only
B. 2 only
C. Both 1 and 2
D. Neither 1 nor 2

prelims.insightsonindia.com 28
© Insights Active Learning | All rights reserved - 131815. You may not reproduce, distribute or exploit the contents in any form without
written permission by copyright owner. Copyright infringers may face civil and criminal liability
Total Marks : 200
Online Prelims TEST - 11 (SUBJECT WISE)
( InsightsIAS Mock Test Series for UPSC Preliminary Exam 2020 ) Mark Scored : 53.33

Your Answer :
Correct Answer : C

Answer Justification :

Both the statements given above are correct.

Subsidies provided by Central Government:

Food > Fertilizer> Petroleum > Interest subsidy > Others.

4
1 03
4 64
5
- 87
.com
a i l
g m
2 2@
am
up
s an
- da
s
Da
m
u pa
An

49 Which of the following is/are the functions of National Statistical Office?


1. It prepares national accounts as well as publishes annual estimates of national product, government
and private consumption expenditure.
2. It organizes and conducts periodic All-India Economic censuses.
3. It releases grants-in-aid to registered non-governmental organizations and research institutions of
repute for undertaking special studies or surveys, printing of statistical reports.

Which of the statements given above is/are correct?


A. 1 and 2 only
B. 1, 2 and 3
C. 2 and 3 only
D. None

Your Answer : B

prelims.insightsonindia.com 29
© Insights Active Learning | All rights reserved - 131815. You may not reproduce, distribute or exploit the contents in any form without
written permission by copyright owner. Copyright infringers may face civil and criminal liability
Total Marks : 200
Online Prelims TEST - 11 (SUBJECT WISE)
( InsightsIAS Mock Test Series for UPSC Preliminary Exam 2020 ) Mark Scored : 53.33

Correct Answer : B

Answer Justification :

NSO is mandated with the following responsibilities:

acts as the nodal agency for planned development of the statistical system in the country, lays
down and maintains norms and standards in the field of statistics, involving concepts and
definitions, methodology of data collection, processing of data and dissemination of results;

coordinates the statistical work in respect of the Ministries/Departments of the Government of


India and State Statistical Bureaus (SSBs), advises the Ministries/Departments of the
Government of India on statistical methodology and on statistical analysis of data;

prepares national accounts as well as publishes annual estimates of national product,


government and private consumption expenditure, capital formation, savings,4estimates of
3
capital stock and consumption of fixed capital, as also the state level gross0capital formation
41
of supra-regional sectors and prepares comparable estimates of State
4 6 Domestic Product (SDP)
at current prices; 7 5
-8
.c om
i l
maintains liaison with international statistical organizations,
a
such as, the United Nations
Statistical Division (UNSD), the Economic and
g mSocial Commission for Asia and the Pacific
(ESCAP), the Statistical Institute for Asia@and the Pacific (SIAP), the International Monetary
2 2 (ADB), the Food and Agriculture Organizations
Fund (IMF), the Asian Development m Bank
(FAO), the International LabourpaOrganizations (ILO), etc.
u
n
a sa
- d
a s
compiles and releases the Index of Industrial Production (IIP) every month in the form of
D
‘quick estimates’; conducts the Annual Survey of Industries (ASI); and provides statistical
m
a to assess and evaluate the changes in the growth, composition and structure of
information
u p
An
the organized manufacturing sector;

organizes and conducts periodic all-India Economic Censuses and follow-up enterprise
surveys, provides an in-house facility to process the data collected through various socio
economic surveys and follow-up enterprise surveys of Economic Censuses;

conducts large scale all-India sample surveys for creating the database needed for studying
the impact of specific problems for the benefit of different population groups in diverse socio
economic areas, such as employment, consumer expenditure, housing conditions and
environment, literacy levels, health, nutrition, family welfare, etc;

examines the survey reports from the technical angle and evaluates the sampling design
including survey feasibility studies in respect of surveys conducted by the National Sample
Survey Organizations and other Central Ministries and Departments;

prelims.insightsonindia.com 30
© Insights Active Learning | All rights reserved - 131815. You may not reproduce, distribute or exploit the contents in any form without
written permission by copyright owner. Copyright infringers may face civil and criminal liability
Total Marks : 200
Online Prelims TEST - 11 (SUBJECT WISE)
( InsightsIAS Mock Test Series for UPSC Preliminary Exam 2020 ) Mark Scored : 53.33

dissemination of statistical information on various aspects through a number of publications


distributed to Government, semi-Government, or private data users/ agencies; and
disseminates data, on request, to the United Nations agencies like the UNSD, the ESCAP, the
ILO and other international agencies;

releases grants-in-aid to registered Non-Governmental Organizations and research


institutions of repute for undertaking special studies or surveys, printing of statistical reports,
and financing seminars, workshops and conferences relating to different subject areas of
official statistics.

http://www.mospi.gov.in/about-ministry-0

50 Which of the following statements best describes ‘Fragile Five’?


0 34
A. Emerging market economies that have become too dependent on unreliable 6 41 foreign
investment to finance their growth ambitions. 7 54
B. Developed western countries whose financial status become
8 to due global slowdown.
- worst
C. Underdeveloped countries in the Sub Saharan region
. c oofma Africa where internal conflicts
crippled the growth prospects. a il
D. None of the statements (a), (b) and (c) are g m
correct.
2 @
m 2
pa
Your Answer :
Correct Answer : A
n u
a sa
Answer Justification :- d
D as
am
https://www.livemint.com/Industry/pAniNfeFWLREH9MrDxfzGL/Indias-journey-from-Fragile-Five-to
u p
-favoured-investment-de.html
An
51 Consider the following statements
1. PSBloansin59minutes.com will enable in principle approval for MSME loans up to Rs. 1 crore within
59 minutes from RBI and 5 Public Sector Banks (PSBs).
2. Department of Financial Services (DFS), Ministry of Finance and National Informatics Centre (NIC)
has jointly developed a mobile app called Jan Dhan Darshak as a part of financial inclusion (FI)
initiative.

Which of the statements given above is/are correct?


A. 1 only
B. 2 only
C. Both 1 and 2
D. Neither nor 2

Your Answer : B
Correct Answer : B

prelims.insightsonindia.com 31
© Insights Active Learning | All rights reserved - 131815. You may not reproduce, distribute or exploit the contents in any form without
written permission by copyright owner. Copyright infringers may face civil and criminal liability
Total Marks : 200
Online Prelims TEST - 11 (SUBJECT WISE)
( InsightsIAS Mock Test Series for UPSC Preliminary Exam 2020 ) Mark Scored : 53.33

Answer Justification :

PSBloansin59minutes.com will enable in principle approval for MSME loans up to Rs. 1 crore within
59 minutes from SIDBI and 5 Public Sector Banks (PSBs). Hence Statement 1 is incorrect.

Department of Financial Services (DFS), Ministry of Finance and National Informatics Centre (NIC)
has jointly developed a mobile app called Jan Dhan Darshak as a part of financial inclusion (FI)
initiative. Hence Statement 2 is correct.

52 In the context of which of the following do you sometimes find the terms ‘amber box, blue box and
green box’ in the news?

A. WTO affairs
B. SAARC affairs
C. UNFCCC affairs
D. India-EU negotiations on FTA 4
1 03
Your Answer : A 4 64
5
Correct Answer : A
- 87
Answer Justification :
.com
a i l
gm by “Boxes” which are given the colours of
In WTO terminology, subsidies in general are identified
@
traffic lights: green (permitted), amber (slow2down — i.e. be reduced), red (forbidden).
a m2
p complicated. The Agriculture Agreement has no Red Box,
In agriculture, things are, as usual,umore
n
a sa
although domestic support exceeding the reduction commitment levels in the Amber Box is
prohibited; and there is -a d
Blue Box for subsidies that are tied to programmes that limit production.
s
Da
There are also exemptions for developing countries

p am
https://www.wto.org/english/tratop_e/agric_e/negs_bkgrnd13_boxes_e.htm
u
An
53 The sale and transport of explosive materials is regulated by the Petroleum and Explosives Safety
Organization (PESO). It works under which of the following ministry?

A. Ministry of Petroleum and Natural gas.


B. Ministry of Commerce and Trade
C. Ministry of Defence
D. Ministry of Chemical and Fertilizers

Your Answer : B
Correct Answer : B

Answer Justification :

Petroleum And Explosives Safety Organisation (PESO) is the department formed by Government of
India to control and administer the usage of explosives, petrol stations in India. It functions under

prelims.insightsonindia.com 32
© Insights Active Learning | All rights reserved - 131815. You may not reproduce, distribute or exploit the contents in any form without
written permission by copyright owner. Copyright infringers may face civil and criminal liability
Total Marks : 200
Online Prelims TEST - 11 (SUBJECT WISE)
( InsightsIAS Mock Test Series for UPSC Preliminary Exam 2020 ) Mark Scored : 53.33

Department Of Industrial Policy and Promotion (DIPP), Ministry of Commerce and Trade.

54 Consider the following statements regarding the Buffer Stock Management in India?
1. At present, buffer stock is maintained for rice and wheat only.
2. Ministry of Agriculture fixes the minimum buffer norms every year.

Which of the statements given above is/are correct?


A. 1 only
B. 2 only
C. Both 1 and 2
D. Neither 1 nor 2

Your Answer : B
Correct Answer : D

4
Answer Justification :
1 03
Both the statements given above are incorrect. 4 64
5
87
Buffer stock is maintained for pulses also. Cabinet committee -on economic affairs fixes buffer
m
norms in every year. .co il
m a
The Cabinet Committee on Economic Affairs fixesgthe minimum buffer norms on quarterly basis: i.e
@ of every financial year. The latest norms set
2January
2
as on 1st April, 1st July, 1st October and 1st
may be seen here. On 15 December 2015,
p amit was decided by the Government to create a buffer stock
u
an Marketing Federation of India Limited (NAFED), Small Farmers
of pulses of 1.5 lakh tonnes to control fluctuation of prices of pulses. Government has engaged
a s
National Agricultural Cooperative
Agri-business Consortium - d(SFAC) and Food Corporation of India (FCI) to procure pulses for buffer
a s
stock. D
am
up
http://www.arthapedia.in/index.php?title=Foodgrain_Management_in_India
An
55 Consider the following statements:
1. Cabinet committee of economic affairs is headed by Finance minister.
2. Cabinet committee on Investment and Growth is headed by Minister of Trade and Commerce.

Which of the statements given above is/are correct?


A. 1 only
B. 2 only
C. Both 1 and 2
D. Neither 1 nor 2

Your Answer : D
Correct Answer : D

Answer Justification :

prelims.insightsonindia.com 33
© Insights Active Learning | All rights reserved - 131815. You may not reproduce, distribute or exploit the contents in any form without
written permission by copyright owner. Copyright infringers may face civil and criminal liability
Total Marks : 200
Online Prelims TEST - 11 (SUBJECT WISE)
( InsightsIAS Mock Test Series for UPSC Preliminary Exam 2020 ) Mark Scored : 53.33

Both the statements are incorrect.

Cabinet committee on economic affairs is headed by Prime Minister.

Cabinet committee on Investment and Growth is headed by Prime Minister.

https://pib.gov.in/newsite/PrintRelease.aspx?relid=190306

56 Consider the following statements:


1. The National Planning Committee was set up in 1938 under the chairmanship of Subhash Chandra
Bose.
2. In 1944, British government created a Planning and Development department under a separate
member of Viceroy’s executive council.

Which of the statements given above is/are correct?


A. 1 only 4
B. 2 only 1 03
C. Both 1 and 2 4 64
5
D. Neither 1 nor 2
- 87
Your Answer : C
.com
Correct Answer : B a i l
g m
2@
m2
Answer Justification :
a
In October 1938, at a conference u ofpMinisters of Industries held under the Chairmanship of
n
Shri Subhash Chander Bose,
a sathe then President of the Indian National Congress, a resolution was
passed which stated “the
s -dproblems such as of poverty and unemployment, of national Defense and
D a
of economic regeneration in general cannot be solved without industrialization. As a step towards

p am
such industrialization, a comprehensive scheme of National Planning should be formulated.”

This A nufollowed by the formulation of “National Planning Committee’ under the


was
Chairmanship of Pandit Jawahar Lal Nehru. The committee consisted of 15 members. Hence,
statement 1 is incorrect.

In 1944 that the British government created a Planning and Development Department
under a separate member of the Viceroy’s Executive Council for organizing and co-
coordinating economic planning in India. Hence, statement 2 is correct.

57 Consider the following statements:


1. M.N. Roy formulated the Gandhian Plan in 1944.
2. The People’s plan was formulated by Narayan Agarwal in 1945.
3. The Sarvodaya plan was formulated by the famous socialist leader Jayaprakash Narayan.

Which of the statements given above is/are correct?


A. 3 only
B. 1 and 2 only

prelims.insightsonindia.com 34
© Insights Active Learning | All rights reserved - 131815. You may not reproduce, distribute or exploit the contents in any form without
written permission by copyright owner. Copyright infringers may face civil and criminal liability
Total Marks : 200
Online Prelims TEST - 11 (SUBJECT WISE)
( InsightsIAS Mock Test Series for UPSC Preliminary Exam 2020 ) Mark Scored : 53.33

C. 1, 2 and 3
D. 2 and 3 only

Your Answer :
Correct Answer : A

Answer Justification :

Keeping the spirit of the Gandhian economic thinking, Sriman Narayan Agarwal formulated the
Gandhian Plan in 1944 which laid more emphasis on agriculture. It also promoted the rural and
cottage industries. Hence, statement 1 is incorrect.

The People’s plan was formulated by M.N. Roy. The plan was based on Marxist socialism and
advocated the need of providing the people with the ‘basic necessities of life’. Hence, statement 2
is incorrect.

The Sarvodaya plan was formulated by the famous socialist leader Jayaprakash
0 34 Narayan.
1 works by the
The plan drew its major inspirations from the Gandhian techniques of constructive
community and trusteeship as well as the Sarvodaya concept of Acharya4 64 Bave. Hence,
Vinoba
5
statement 3 is correct. 87 -
.c om
58 With reference to the five year planning in India, whichaofilthe following statements is/are correct?
gm on development of agriculture.
1. The first five year plan (1951-56) laid highest priority
@
2
2. There were annual for three consecutive years 2 between the second and third five year plans.
ameleventh five year plan.
3. Indicative planning in India started from
p
u
an
as the code given below:
Select the correct answer using
d
A. 1 only
s -
B. 1 and 3 onlyD a
C. 1 and a2monly
p
u 3 only
D. 2 and
An
Your Answer :
Correct Answer : A

Answer Justification :

The First Five-year Plan was launched in 1951 which mainly focused in development of the primary
sector. The total planned budget of Rs.2069 crore (2378 crore later) was allocated to seven broad
areas: irrigation and energy (27.2%), agriculture and community development
(17.4%), transport and communications (24%), industry (8.4%), social
services (16.6%), rehabilitation of landless farmers (4.1%), and for other sectors
and services (2.5%). Hence, statement 1 is correct.

The period of the three consecutive Annual Plans was 1966–69. It was the period between the third
and fourth five year plans. Hence, statement 2 is incorrect.

In indicative Planning, the planning authority sets the broad goals for the economy. The

prelims.insightsonindia.com 35
© Insights Active Learning | All rights reserved - 131815. You may not reproduce, distribute or exploit the contents in any form without
written permission by copyright owner. Copyright infringers may face civil and criminal liability
Total Marks : 200
Online Prelims TEST - 11 (SUBJECT WISE)
( InsightsIAS Mock Test Series for UPSC Preliminary Exam 2020 ) Mark Scored : 53.33

implementation of the plan requires the co-ordination of the various units in the economy. This
type of planning in India since the 8th Five-Year Plan. Hence, statement 3 is incorrect.

59 With reference to the economic critique of British rule, consider the


following pairs:
Personality: Work:
1. Dadabhai Naoroji Poverty and Un-British Rule in India
2. R.C. Dutt India Today
3. R. P. Dutt Economic History of India

Which of the pairs given above is/are correctly matched?


A. 1 only
B. 1 and 3 only
C. 2 and 3 only
D. 1, 2 and 3 4
1 03
Your Answer : B 4 64
5
Correct Answer : A
- 87
Answer Justification : .com
a i l
m
Dadabhai Naoroji wrote the book ‘Poverty andgUn-British Rule in India’. It discussed how the
2 @
2
government was destructive and despotic to the Indians and un-British and suicidal to
am
Britain. Hence, pair 1 is correctly matched.
n up
The book ‘Economic Historysa
incorrectly matched. - d
a of India’ was written by Romesh Chandra Dutt. Hence, pair 2 is
D as
m Today’ was written by written by R. Palme Dutt. Hence, pair 3 is incorrectly
The book ‘India
a
matched.p
u
An
60 Consider the following statements:
1. Lord Curzon passed Co-operative Society Act in1904.
2. In 1914 the Government appointed a committee under the chairmanship of Sir Edward Mac Lagan
to review the progress of co-operative movement.
3. Under the Montague- Chelmsford Act of 1919, co-operation became a provincial subject.

Which of the statements given above is/are correct?


A. 1 and 3 only
B. 1 and 2 only
C. 1, 2 and 3
D. 2 and 3 only

Your Answer :
Correct Answer : C

prelims.insightsonindia.com 36
© Insights Active Learning | All rights reserved - 131815. You may not reproduce, distribute or exploit the contents in any form without
written permission by copyright owner. Copyright infringers may face civil and criminal liability
Total Marks : 200
Online Prelims TEST - 11 (SUBJECT WISE)
( InsightsIAS Mock Test Series for UPSC Preliminary Exam 2020 ) Mark Scored : 53.33

Answer Justification :

The Famine Commission of 1901 strongly recommended that in order to prevent famine,
agriculturists should be granted loans to improve agriculture. By 1904, the Co-operative Society
Act was passed by the Viceroy Lord Curzon. Hence, statement 1 is correct.

In 1914 the Government appointed a committee under the chairmanship of Sir Edward Mac
Lagan to review the progress of co-operative movement.

The committee observed that illiteracy and ignorance of the masses, misappropriation of funds,
rampant nepotism, inordinate delay in granting loans and viewing co-operative movement as a
government movement were some of the glaring defects of the co-operative movement.

Hence, statement 2 is correct.

Under the Montague- Chelmsford Act of 1919, co-operation became a provincial subject
which gave further impetus to the movement. Various states passed their own 4 Acts to make co-
3
10 increased
operative movement a successful one. The membership of the co-operative societies
4
considerably during this period. Hence, statement 3 is correct. 6 4
5
- 87
61 Consider the following statements:
. c om
2. Full time factory inspectors were appointed under m
a l 14 years could be employed in a factory.
1. Factory Act, 1881 ruled that no children below the age iof
the Factory Act, 1911.
3. The Whitley Commission on Labour was set up 2 @ing1929 to inquire into the existing conditions
of labour in industrial undertakings andm 2
plantations in India.
u pa
n
Which of the statements given
a saabove is/are correct?
A. 3 only
B. 1 and 2 only as
-d
C. 1, 2 and 3 D
m
D. 2 and
u pa3 only
An
Your Answer :
Correct Answer : D

Answer Justification :

Factories Act of I881 was the first legislation directly affecting industrial labourers in the country.
It applied to all manufacturing establishments using power driven machinery, employing 100 or
more labours and working for more than 120 days in a year. It ruled that no children below the
age of 7 could be employed in a factory and nobody before attaining the age of 12 could be
made to work for more than 9 hours a day. Hence, statement 1 is incorrect.

Full time factory inspectors were appointed under the Factory Act, 1911 to see whether the
provisions were being enforced. Hence, statement 2 is correct.

The Royal Commission on Labour or the Whitley Commission on Labour was set up in 1929 to
inquire into the existing conditions of labour in industrial undertakings and plantations in

prelims.insightsonindia.com 37
© Insights Active Learning | All rights reserved - 131815. You may not reproduce, distribute or exploit the contents in any form without
written permission by copyright owner. Copyright infringers may face civil and criminal liability
Total Marks : 200
Online Prelims TEST - 11 (SUBJECT WISE)
( InsightsIAS Mock Test Series for UPSC Preliminary Exam 2020 ) Mark Scored : 53.33

India. The Commission recommended the adoption of suitable legislation enabling the Competent
Authority to collect and collate information regarding the living, working and socio-economic
conditions of industrial labour. Hence, statement 3 is correct.

https://shodhganga.inflibnet.ac.in/bitstream/10603/89669/7/07_chapter-ii.pdf

62 The Saharanpur Rules of 1855 is related to which of the following subjects?

A. Ryotwari system
B. Permanent Revenue system
C. Mahalwari system
D. Industrial dispute Act, 1927

Your Answer : C
Correct Answer : C
4
1 03
64
Answer Justification :
54
87UP, the Central Provinces the
The Mahalwari tenure was introduced in1822 in major portions of the
-
Punjab and covered nearly 30% of the area.
. c om
The Mahalwari system of land revenue worked underathe il scheme of 1833 was completed under the
administration of James Thompson. The 66% rental
@ gmdemanded proved very harsh, too. In the
Saharanpur Rules of 1855, it was revised to2 2
50% by Lord Dalhousie.
m
Hence, option (c) is correct. u pa
n
a sa
s -d
63 Consider the following statements
DaPlan was largely based on the Mahalanobis model.
1. The First Five-Year
m
aFive-Year
2. The Second
u p Plan was largely based on the Harrod–Domar model.
An
Which of the statements given above is/are correct?
A. 1 only
B. 2 only
C. Both 1 and 2
D. Neither 1 nor 2

Your Answer : A
Correct Answer : D

Answer Justification :

The First Five-year Plan was launched in 1951 which mainly focused in development of
the primary sector. The First Five-Year Plan was based on the Harrod–Domar model with few
modifications. Hence, statement 1 is incorrect.

The Second Plan focused on the development of the public sector and "rapid Industrialisation".

prelims.insightsonindia.com 38
© Insights Active Learning | All rights reserved - 131815. You may not reproduce, distribute or exploit the contents in any form without
written permission by copyright owner. Copyright infringers may face civil and criminal liability
Total Marks : 200
Online Prelims TEST - 11 (SUBJECT WISE)
( InsightsIAS Mock Test Series for UPSC Preliminary Exam 2020 ) Mark Scored : 53.33

The plan followed the Mahalanobis model, an economic development model developed by the
Indian statistician Prasanta Chandra Mahalanobis in 1953. Hence, statement 2 is incorrect.

64 Consider the following statements:


1. The twenty-point programme was launched during the Fifth five year plan.
2. The National Bank for Agriculture and Rural Development (NABARD) was established for
development of rural areas on the recommendation of the Shivaraman Committee.

Which of the statements given above is/are correct?


A. 1 only
B. 2 only
C. Both 1 and 2
D. Neither 1 nor 2

Your Answer :
4
Correct Answer : C
1 03
4 64
Answer Justification : 5
- 87
The twenty-point programme was launched during the Fifth five
o m year plan in 1975 by the then
. c
Prime minister Indira Gandhi. The basic objective of thel20-Point
i Programme is to eradicate poverty
a privileged population of the country.
and to improve the quality of life of the poor and the under
m
g
2@
m2
Hence, statement 1 is correct.
a
National Bank for Agriculturenand up Rural Development (NABARD) is an apex development
sais an institution fully owned by Government of India. NABARD was
financial institution in India.aIt

s -d
established on the recommendations of B.Sivaraman Committee, (by Act 61, 1981 of Parliament)
a
D the National Bank for Agriculture and Rural Development Act 1981. It
in 1982 to implement
a m
replaced the Agricultural Credit Department (ACD) and Rural Planning and Credit Cell (RPCC)
p
u Bank of India, and Agricultural Refinance and Development Corporation (ARDC).
of Reserve
An
Hence, statement 2 is correct.

65 Bureau of Energy Efficiency star libeling is applied to which of the following appliances
1. Tubular Florescent Lamp
2. LPG- Stoves
3. Distribution Transformer
4. Agricultural Pumpset

Select the correct answer using the code given below


A. 1 and 2 only
B. 2 and 3 only
C. 1, 2 and 3 only
D. 1, 2, 3, 4

Your Answer : D

prelims.insightsonindia.com 39
© Insights Active Learning | All rights reserved - 131815. You may not reproduce, distribute or exploit the contents in any form without
written permission by copyright owner. Copyright infringers may face civil and criminal liability
Total Marks : 200
Online Prelims TEST - 11 (SUBJECT WISE)
( InsightsIAS Mock Test Series for UPSC Preliminary Exam 2020 ) Mark Scored : 53.33

Correct Answer : D

Answer Justification :

BEE star labelling is currently invoked for equipments/appliances Room Air Conditioner (Fixed
Speed), Ceiling Fan, Colour Television, Computer, Direct Cool Refrigerator, Distribution
Transformer, Domestic Gas Stove, Frost Free Refrigerator, General Purpose Industrial Motor,
Monoset Pump, Openwell Submersible Pump Set, Stationary Type Water Heater, Submersible Pump
Set, Tfl, Washing Machine, Ballast, Solid State Inverter, Office Automation Products, Diesel Engine
Driven Monosetpumps For Agricultural Purposes, Diesel Generator Set, Led Lamps, Room Air
Conditioner (Variable Speed), Chillers, Agricultural Pumpset.

Hence, option (d) is correct.

66 “Online Portal Ensure”, sometime seen in the news, is related to


4
A. Procurement of goods by government departments 1 03
B. Animal Husbandry 4 64
5
C. Procurement of Pulses
- 87
D. Citizen Charter
.com
Your Answer : a i l
Correct Answer : B g m
2 2@
Answer Justification : am
n up
Online Portal “ENSURE” asa

s -d
D a
Union Minister of Agriculture and Farmers’ Welfare launched a portal ENSURE – National
m
Livestock Mission-EDEG
aDairying developed by NABARD and operated under the Department of Animal
u
Husbandry,p & Fisheries.
An
Entrepreneurship Development and Employment Generation (EDEG):

Under the Mission’s component EDEG, subsidy payment for activities related to poultry, small
ruminants, pigs etc., through Direct Benefit Transfer (DBT) goes directly to the beneficiary’s
account.

To make it better, simpler and transparent, the NABARD has developed an online portal
“ENSURE” which makes the information related to beneficiary and processing of application
readily available.

Hence, option (b) is correct.

67 Consider the following statements regarding Institutes of Eminence Scheme.

prelims.insightsonindia.com 40
© Insights Active Learning | All rights reserved - 131815. You may not reproduce, distribute or exploit the contents in any form without
written permission by copyright owner. Copyright infringers may face civil and criminal liability
Total Marks : 200
Online Prelims TEST - 11 (SUBJECT WISE)
( InsightsIAS Mock Test Series for UPSC Preliminary Exam 2020 ) Mark Scored : 53.33

1. It was rolled out by NITI Ayog.


2. It aims to project Indian institutes to global recognition.
3. Selected institutes under this scheme will enjoy complete academic and administrative autonomy.

Which of the statements given above is/are correct?


A. 1 and 2 only
B. 2 only
C. 2 and 3 only
D. 1, 2 and 3

Your Answer : C
Correct Answer : C

Answer Justification :

Institutions of Eminence Scheme


0 34
The institutes of eminence scheme under the Union human resource development
6 41 (HRD) ministry
aims to project Indian institutes to global recognition.
7 54
8
- (UGC).
The scheme was rolled out by University Grants Commission
o m Hence, statement 1 is
incorrect.
il. c
a
It aimed to help 20 higher education (10 public
@ gmand 10 private) institutions from country
2 2
break into top 500 in at least one internationally reputed ranking framework in 10 years,
m time. Hence, statement 2 is correct.
and then eventually break into top 100aover
n up
a sa complete academic and administrative autonomy. Hence,
The selected institutes will enjoy
statement 3 is correct.
s -d
D a
am except institutions in MEA and MHA’s list of negative countries.
They will be exempted from approvals of government or UGC for academic collaborations with
u p
foreign institutions,
An
68 Consider the following statements regarding National Defence Fund
1. It is administered by an Executive Committee, with PM as Chairperson.
2. The fund is entirely dependent on voluntary contributions from the public and does not get any
budgetary support.

Which of the statements given above is/are correct?


A. 1 only
B. 2 only
C. Both 1 and 2
D. Neither 1 nor 2

Your Answer : C
Correct Answer : C

Answer Justification :

prelims.insightsonindia.com 41
© Insights Active Learning | All rights reserved - 131815. You may not reproduce, distribute or exploit the contents in any form without
written permission by copyright owner. Copyright infringers may face civil and criminal liability
Total Marks : 200
Online Prelims TEST - 11 (SUBJECT WISE)
( InsightsIAS Mock Test Series for UPSC Preliminary Exam 2020 ) Mark Scored : 53.33

It is used for the welfare of the members of the Armed Forces (including Para Military Forces)
and their dependents

The Fund is administered by an Executive Committee, with PM as Chairperson, and


Defence, Finance and Home Ministers as Members. Hence, statement 1 is correct.

Finance Minister is the Treasurer of the Fund. Accounts of the Fund are kept with the Reserve Bank
of India.

The fund is entirely dependent on voluntary contributions from the public and does not get any
budgetary support. Hence, statement 2 is correct.

69 Consider the following statements regarding Transformation of Aspirational Districts.


1. It aims to quickly and effectively transform some of India's most underdeveloped districts.
2. It aims to develop around 500 districts under developed districts by 2022.
4 become the
3. Officers at the level of Joint Secretary / Additional Secretary have been nominated to
3
'Central Prabhari Officers' of each district. 10 4
6
Which of the statements given above is/are correct? 7 54
A. 1 only -8
B. 2 and 3 only
. c om
C. 1 and 3 only a il
D. 1, 2 and 3 gm @
2 2
m
Your Answer : D
u pa
Correct Answer : C
a n
s
Answer Justification :-
da
D as
m
The ‘Transformation
a of Aspirational Districts’ programme aims to quickly and effectively
p
u these districts. Hence, statement 1 is correct.
transform
An
The broad contours of the programme are Convergence (of Central & State Schemes), Collaboration
(of Central, State level ‘Prabhari’ Officers & District Collectors), and Competition among districts
driven by a mass Movement. Hence, statement 3 is correct.

With States as the main drivers, this program will focus on the strength of each district, identify
low-hanging fruits for immediate improvement, measure progress, and rank districts.

The 112 districts were identified from 27 states, at least one from each state, in a
transparent manner by a committee of Senior Officers to the Government of India, in
consultation with State Officials using a composite index of key data sets that included deprivation
enumerated under the Socio-Economic Caste Census, key health and education sector performance
and state of basic infrastructure. Hence, statement 2 is incorrect.

https://niti.gov.in/about-aspirational-districts-programme

http://vikaspedia.in/social-welfare/rural-poverty-alleviation-1/transformation-of-aspirational-districts

prelims.insightsonindia.com 42
© Insights Active Learning | All rights reserved - 131815. You may not reproduce, distribute or exploit the contents in any form without
written permission by copyright owner. Copyright infringers may face civil and criminal liability
Total Marks : 200
Online Prelims TEST - 11 (SUBJECT WISE)
( InsightsIAS Mock Test Series for UPSC Preliminary Exam 2020 ) Mark Scored : 53.33

70 Amended Technology Upgradation Fund Scheme (ATUFS), sometime seen in the news, is related to

A. Textile Sector
B. Startup India
C. Agro-Industries
D. None of the above

Your Answer : A
Correct Answer : A

Answer Justification :

It is a credit-linked Central Sector Scheme.

There is a provision of one-time capital subsidy for eligible benchmarked machinery at the rate
of 15% for garmenting and technical textiles segments with a cap of Rs. 30 crore and at
4 a cap of Rs.
3with
the rate of 10% for weaving, processing, jute, silk and handloom segments
1 0
20 crore. 64 4
5
Hence, option (a) is correct. - 87
om
a
71 With reference to the India’s Foreign Trade Policy (FTP)
l.c
i2015-20, consider the following statements:
@ gm Exports from India Scheme (MEIS)” and
1. It introduced two new schemes, namely “Merchandise
2
“Services Exports from India Scheme (SEIS)”. 2
am from 2 per cent to 5 per cent.
pexports
2. It aims to raise India’s share in world
u
an EPCG scheme to 75% to promote domestic capital goods
3. It reduced export obligation under
manufacturing. as d
-
D
Which of the statements as
given above is/are correct?
A. 1 onlya m
up 3 only
B. 1nand
C.A1, 2 and 3
D. 2 and 3 only

Your Answer :
Correct Answer : B

Answer Justification :

India’s Foreign Trade Policy 2015-20 provides a framework for increasing exports of goods and
services as well as generation of employment and increasing value addition in the country, in line
with the ‘Make in India’ programme.

FTP 2015-20 introduces two new schemes, namely ‘Merchandise Exports from India Scheme
(MEIS)’ for export of specified goods to specified markets and ‘Services Exports from India
Scheme (SEIS)’ for increasing exports of notified services. Duty credit scrips issued under
MEIS and SEIS and the goods imported against these scrips are fully transferable.

prelims.insightsonindia.com 43
© Insights Active Learning | All rights reserved - 131815. You may not reproduce, distribute or exploit the contents in any form without
written permission by copyright owner. Copyright infringers may face civil and criminal liability
Total Marks : 200
Online Prelims TEST - 11 (SUBJECT WISE)
( InsightsIAS Mock Test Series for UPSC Preliminary Exam 2020 ) Mark Scored : 53.33

Hence, statement 1 is correct.

It aims to raise India’s share in world exports from 2 per cent to 3.5 per cent. Hence,
statement 2 is incorrect.

Measures have been adopted to nudge procurement of capital goods from indigenous
manufacturers under the EPCG scheme by reducing specific export obligation to 75per
cent of the normal export obligation. Hence, statement 3 is correct.

https://www.ibef.org/pages/foreign-trade-policy-2015-20-key-highlights

72 With reference to the National Policy on Software Products – 2019, consider the following
statements:
1. It aims to a ten-fold increase in share of the Global Software product market by 2025.
2. It aims to nurture 10,000 technology startups in software product industry in Tier-II and Tier-III
towns and cities. 4 3
10
Which of the statements given above is/are correct?
4 64
5
A. 1 only
B. 2 only - 87
C. Both 1 and 2
.com
D. Neither 1 nor 2 a i l
g m
Your Answer : C
2 2@
Correct Answer : A am
up
s an
da
Answer Justification :
-
as
To create a robust software
D product ecosystem the Government has approved the National Policy
on Software m Products - 2019, which aims to develop India as the global software product hub,
p a
n u
driven by innovation, improved commercialization, sustainable Intellectual Property (IP), promoting
A
technology startups and specialized skill sets.

It aims to promote the creation of a sustainable Indian software product industry, driven by
intellectual property (IP), leading to a ten-fold increase in share of the Global Software
product market by 2025. Hence, statement 1 is correct.

It aims to nurture 10,000 technology startups in software product industry, including 1000 such
technology startups in Tier-II and Tier-III towns & cities and generating direct and in-direct
employment for 3.5 million people by 2025. Hence, statement 2 is incorrect.

http://vikaspedia.in/e-governance/digital-india/national-policy-on-software-products-2019

73 With reference to the Renewable energy sector in India, consider the following statements:
#94000
1. Solar capacity has increased by eight times during the period 2014-18.
2. Inter-state distribution of wind power was started in 2018.
3. World’s largest solar park named ‘Shakti Sthala’ was launched in Rajasthan.
prelims.insightsonindia.com 44
© Insights Active Learning | All rights reserved - 131815. You may not reproduce, distribute or exploit the contents in any form without
written permission by copyright owner. Copyright infringers may face civil and criminal liability
Total Marks : 200
Online Prelims TEST - 11 (SUBJECT WISE)
( InsightsIAS Mock Test Series for UPSC Preliminary Exam 2020 ) Mark Scored : 53.33

Which of the statements given above is/are correct?


A. 2 only
B. 1 and 2 only
C. 1, 2 and 3
D. 2 and 3 only

Your Answer : C
Correct Answer : B

Answer Justification :

According to Renewable Energy Country Attractiveness index 2018, the Indian renewable energy
sector is the fourth most attractive renewable energy market in the world. As of October 2018,
India ranked 5th in installed renewable energy capacity. According to 2018 Climatescope report
India ranked second among the emerging economies to lead to transition to clean energy.

34 11,788 MW
Solar capacity has increased by eight times between FY14-18. India added record
0
of renewable energy capacity in 2017-18. Hence, statement 1 is correct. 41
5 46
Inter-state distribution of wind power was started in August 8 7 Hence, statement 2 is
2018.
-
om
correct.
l . c
ilaunched
World’s largest solar park named ‘Shakti Sthala’ was
m a in Karnataka in March 2018
with an investment of Rs 16,500 crores. Hence, g statement 3 is incorrect.
2 2@
m
pa
https://www.ibef.org/industry/renewable-energy.aspx
u
n
a sa
- d Manufacturing Policy, consider the following statements:
74 With reference to the National
s
Damillion additional jobs by 2022.
1. It aims to increase the sectoral share of manufacturing in GDP to at least 20% by 2022.

am
2. It aims to create 10
u p
WhichAnof the statements given above is/are correct?
A. 1 only
B. 2 only
C. Both 1 and 2
D. Neither 1 nor 2

Your Answer : C
Correct Answer : D

Answer Justification :

The major objectives of the National Manufacturing Policy are to increase the sectoral share of
manufacturing in GDP to at least 25% by 2022; to increase the rate of job creation so as to
create 100 million additional jobs by 2022; and to enhance global competitiveness, domestic
value addition, technological depth and environmental sustainability of growth.

Hence, both statement 1 and statement 2 are incorrect.

prelims.insightsonindia.com 45
© Insights Active Learning | All rights reserved - 131815. You may not reproduce, distribute or exploit the contents in any form without
written permission by copyright owner. Copyright infringers may face civil and criminal liability
Total Marks : 200
Online Prelims TEST - 11 (SUBJECT WISE)
( InsightsIAS Mock Test Series for UPSC Preliminary Exam 2020 ) Mark Scored : 53.33

https://www.meity.gov.in/writereaddata/files/National%20Manufacturing%20Policy%20(2011)%20(
167%20KB).pdf

75 In which of the following sectors the government has allowed 100 per cent Foreign Direct Investment
(FDI)?
1. Food Processing
2. Multi-brand Retail Trading
3. Petroleum Refining by PSUs

Select the correct answer using the code given below:


A. 1 and 3 only
B. 1 only
C. 2 and 3 only
D. 1, 2 and 3

4
03
Your Answer : B
1
64
Correct Answer : B
5 4
Answer Justification :
- 87
om
.croute in Food Processing sector.
Government has approved 100 % FDI through automatic
a il
g
Government has approved 49 % FDI through Government m
route in Multi-brand Retail Trading.
2 2@
m automatic route in Petroleum Refining by PSUs.
pa
Government has approved 49 % FDI through
n u
Hence, option (b) is correct.
a sa
s -d
Da
https://www.investindia.gov.in/foreign-direct-investment

p am
u
76 ‘HiLumi’,nrecently in the news, is related to
A
A. EESL’s Street Lighting National Programme
B. CERN’s Large Hadron Collider
C. PETA’s Animal Ethics Initiative
D. NASA’s Black Marble Project

Your Answer : A
Correct Answer : B

Answer Justification :

What is HiLumi?

The High Luminosity LHC (HL-LHC) is an upgrade of the LHC. HiLumi will make it possible to study
the fundamental building blocks of matter in more detail than ever before.

The upgrade will boost the accelerator's potential for new discoveries in physics, starting in 2026.
prelims.insightsonindia.com 46
© Insights Active Learning | All rights reserved - 131815. You may not reproduce, distribute or exploit the contents in any form without
written permission by copyright owner. Copyright infringers may face civil and criminal liability
Total Marks : 200
Online Prelims TEST - 11 (SUBJECT WISE)
( InsightsIAS Mock Test Series for UPSC Preliminary Exam 2020 ) Mark Scored : 53.33

https://hilumilhc.web.cern.ch/

77 The world’s first hydrogen-powered train is rolled out by

A. China
B. Japan
C. South Korea
D. Germany

Your Answer : D
Correct Answer : D

Answer Justification :

Germany has rolled out the world’s first hydrogen-powered train.


4
How do they operate? 1 03
4 64
Hydrogen trains are equipped with fuel cells that produce electricity7 5
through a combination of
- 8
hydrogen and oxygen, a process that leaves steam and water as the only emissions. Excess energy
m
is stored in ion lithium batteries on board the train co il .
m a
g
https://www.indiatoday.in/education-today/gk-current-affairs/story/worlds-first-hydrogen-powered-tr
ain-1342745-2018-09-18 2@ 2
am
n up
sa regarding the findings of All India Rural Financial Inclusion Survey
78 Consider the following statements
a
- d households in the country have outstanding debt.
2016-17.
s
1. More than half the agricultural
Da of indebtedness came from those owning more than two hectares of land.
2. The highest incidence
p am
Whichnu
A of the statements given above is/are correct?
A. 1 only
B. 2 only
C. Both 1 and 2
D. Neither 1 nor 2

Your Answer : C
Correct Answer : C

Answer Justification :

Both the statements are correct

NABARD found that 52.5% of the agricultural households had an outstanding loan on the
date of the survey, and thus were considered indebted.

While all classes of farmers had debt, the highest incidence of indebtedness came from those

prelims.insightsonindia.com 47
© Insights Active Learning | All rights reserved - 131815. You may not reproduce, distribute or exploit the contents in any form without
written permission by copyright owner. Copyright infringers may face civil and criminal liability
Total Marks : 200
Online Prelims TEST - 11 (SUBJECT WISE)
( InsightsIAS Mock Test Series for UPSC Preliminary Exam 2020 ) Mark Scored : 53.33

owning more than two hectares of land. In that category, 60% of households are in debt.

The biggest reason for taking loans among agricultural households was capital expenditure for
agricultural purposes, with a quarter of all loans taken for this purpose.

https://www.google.com/search?q=All+India+Rural+Financial+Inclusion+Survey+2016-17&rlz=1
C1CHBF_enIN732IN732&oq=All+India+Rural+Financial+Inclusion+Survey+2016-17&aqs=chrom
e..69i57.816j0j4&sourceid=chrome&ie=UTF-8

79 Identify the national park based on following features:


1. It is a part of world’s ‘hottest hotspots’ of biological diversity.
2. It is home to unique, threatened and endangered species like the lion-tailed macaques
3. Rivers Tunga, Bhadra and Netravati, originate from the heart of this National Park.

Select the correct answer using the codes given below.


A. Mudumalai National Park 4
B. Bandipur National Park 1 03
C. Kudremukh National Park 4 64
5
D. Silent Valley National Park
- 87
Your Answer : C .com
Correct Answer : C a i l
g m
2@
m2
Answer Justification :
a
n
The Kudremukh National Park is the upsecond largest Wildlife Protected Area (600.32 km2) belonging
to a tropical wet evergreen a sa of forest in the Western Ghats. Kudremukh National Park is located
type
- d District and Chikkamagaluru districts of the State of Karnataka.
in Dakshina Kannada, Udupi
s
Da
am
https://en.wikipedia.org/wiki/Kudremukh
p
A nu
80 Shangri-La Dialogue’, sometimes seen in the news, is related to

A. South-South co-operation summit between third world countries in the world.


B. Asia's premier defence summit of ministers and delegates.
C. Summit by governors of central Banks of G-20 Nations about global financial stability.
D. None of the statements given above (a), (b) and (c) are correct.

Your Answer :
Correct Answer : B

Answer Justification :

The IISS Shangri-La Dialogue is Asia's premier defence summit, a unique


meeting of ministers and delegates. Each year’s agenda is intentionally wideranging, reflecting the
many defence and security challenges facing a large and diverse region.

prelims.insightsonindia.com 48
© Insights Active Learning | All rights reserved - 131815. You may not reproduce, distribute or exploit the contents in any form without
written permission by copyright owner. Copyright infringers may face civil and criminal liability
Total Marks : 200
Online Prelims TEST - 11 (SUBJECT WISE)
( InsightsIAS Mock Test Series for UPSC Preliminary Exam 2020 ) Mark Scored : 53.33

https://www.iiss.org/events/shangri-la-dialogue/shangri-la-dialogue-2019

81 Consider the following statements regarding Amazon Fund


1. It is a G7 mechanism created to raise donations for non-reimbursable investments in efforts to
prevent, monitor and combat deforestation in Amazon basin.
2. It is managed by Brazilian Development Bank.
3. India contributed significant amount of grants to Amazon Fund

Which of the statements given above is/are correct?


A. 3 only
B. 2 and 3 only
C. 2 only
D. 1 only

Your Answer : B
4
Correct Answer : C
1 03
4 64
Answer Justification : 5
- 87
o m
Amazon Fund is a REDD+ initiative to raise donations for non-reimbursable investments in efforts
c
to prevent, monitor and combat deforestation, as well asl.to promote the preservation and
sustainable use in the Brazilian Amazon. ai
@ gm
2 2
Amazon Fund is managed by BNDES, the Brazilian Development Bank.
m
u pa
n
82 Consider the following statements
a sa
- dof Kerala
1. Periyar is the longest river
s
2. Sabarimala dedicated
D a to Lord Ayyappa is located on banks of Periyar River.
3. Kallada River m
p a discharges into Ashtamudi Lake
u
Anof the statements given above is/are correct?
Which
A. 2 only
B. 1 and 3 only
C. 3 only
D. None

Your Answer : B
Correct Answer : B

Answer Justification :

Periyar  is the longest river in  Kerala.

Sabarimala temple dedicated to Lord Ayyappa is located on the banks of the river Pamba 

Kallada River is a major river discharging into the Ashtamudi Lake. The Kallada river, which
originates near Ponmudi from the Kulathupuzha hills Western Ghats is formed by the confluence of

prelims.insightsonindia.com 49
© Insights Active Learning | All rights reserved - 131815. You may not reproduce, distribute or exploit the contents in any form without
written permission by copyright owner. Copyright infringers may face civil and criminal liability
Total Marks : 200
Online Prelims TEST - 11 (SUBJECT WISE)
( InsightsIAS Mock Test Series for UPSC Preliminary Exam 2020 ) Mark Scored : 53.33

three rivers, viz., Kulathupuzha, Chenthurnipuzha, and Kalthuruthipuzha, and after traversing a
distance of about 121 km (75 mi) through virgin forests finally debouches into the Ashtamudi
wetland at Peringalam near Kollam.

83 Consider the following statements regarding African Development Bank (AfDB)


1. It was established in 1960’s to promote socio-economic development of Africa.
2. India is member country of AfDB

Which of the statements given above is/are correct?


A. 1 only
B. 2 only
C. Both 1 and 2
D. Neither 1 nor 2

Your Answer : A
4
Correct Answer : C
1 03
4 64
Answer Justification : 5
- 87
The African Development Bank Group (AfDB) or Banque Africaine
o m de Développement (BAD) is a
c
. founded in 1964 and comprises three
ail
multilateral development finance institution. The AfDB was
entities: The African Development Bank, the African Development Fund and the Nigeria Trust Fund.
@ gm
The AfDB's mission is to fight poverty and 2 2
improve living conditions on the continent through
m
pa private capital in projects and programs that are likely to
promoting the investment of public and
u
an development of the region.
contribute to the economic and social
s
- da
s
Da
p am
u
An

prelims.insightsonindia.com 50
© Insights Active Learning | All rights reserved - 131815. You may not reproduce, distribute or exploit the contents in any form without
written permission by copyright owner. Copyright infringers may face civil and criminal liability
Total Marks : 200
Online Prelims TEST - 11 (SUBJECT WISE)
( InsightsIAS Mock Test Series for UPSC Preliminary Exam 2020 ) Mark Scored : 53.33

https://en.wikipedia.org/wiki/African_Development_Bank

84 Consider the following pairs


Region in News Located in
1. Herat Pakistan
2. Mekong River South East Asia
3. Golan Heights Middle East

Which of the pairs given above is/are correct?


A. 1 only
B. 2 and 3 only
C. 3 only
D. 1, 2 and 3
4
Your Answer : B 1 03
Correct Answer : B 4 64
5
- 87
Answer Justification :
.c om
Herat and Kandahar are cities located in Afghanistanail
gm
2@ Asia. It is the world's twelfth longest river and
The Mekong is a trans-boundary river in Southeast
2
mlength is 4,350 km, and it drains an area of 795,000 km²,
the seventh longest in Asia. Its estimated
a
u
discharging 475 km³ of water annually.p
n
a sa
The Golan Heights, a rocky
- d plateau in south-western Syria, has a political and strategic significance
s seized the Golan Heights from Syria in the closing stages of the 1967
which belies its size.aIsrael
Six-Day War. D
p am
u
An
85 Consider the following statements regarding Brahmos
1. It is a medium range supersonic ballistic missile
2. It is jointly developed by India and Russia.
3. Brahmos can be launched through air-launched variant

Which of the statements given above is/are correct?


A. 3 only
B. 2 and 3 only
C. 1 and 2 only
D. 1, 2 and 3

Your Answer : D
Correct Answer : B

Answer Justification :

prelims.insightsonindia.com 51
© Insights Active Learning | All rights reserved - 131815. You may not reproduce, distribute or exploit the contents in any form without
written permission by copyright owner. Copyright infringers may face civil and criminal liability
Total Marks : 200
Online Prelims TEST - 11 (SUBJECT WISE)
( InsightsIAS Mock Test Series for UPSC Preliminary Exam 2020 ) Mark Scored : 53.33

The BrahMos (designated PJ-10) is a medium-range ramjet supersonic cruise missile that can be
launched from submarine, ships, aircraft, or land.

It is a joint venture between the Russian Federation's NPO Mashinostroyeniya and India's Defence
Research and Development Organisation (DRDO), who together have formed BrahMos Aerospace

Brahmos can be launched through air-launched variant

https://en.wikipedia.org/wiki/BrahMos

86 Consider the following statements


1. Tea Board of India is responsible for the assignment of certification numbers to exports of certain
tea merchants.
2. Tea Board of India coordinates with research institutes, government bodies to ensure technical
support to the tea trade in the global industry.
3. Assam is the largest producer of tea in India. 4 3
10
Which of the statements given above is/are correct?
4 64
5
A. 1, 2 and 3
B. 2 only - 87
C. 3 only
.com
D. 2 and 3 only a i l
g m
Your Answer : A
2 2@
Correct Answer : A am
up
s an
da
Answer Justification :
-
s responsible for the assignment of certification numbers to exports of certain
The Tea Board Indiaais
D
mThis certification is intended to ensure the teas’ origin, which in turn would reduce
tea merchants.
a
upof fraudulent labelling on rare teas such as ones harvested in Darjeeling.
the amount
An
It coordinates research institutes, the tea trade and government bodies, ensuring the technical
support of the tea trade in the global industry.

Assam is the largest producer of tea in India. India is one of the largest producers of tea in the
world, second only to China.

https://en.wikipedia.org/wiki/Tea_Board_of_India

87 Management Effectiveness Evaluation (MEE) report, sometimes seen in the news, is related to

A. Infrastructure Development
B. Border Management
C. Wildlife Conservation
D. None of the above

prelims.insightsonindia.com 52
© Insights Active Learning | All rights reserved - 131815. You may not reproduce, distribute or exploit the contents in any form without
written permission by copyright owner. Copyright infringers may face civil and criminal liability
Total Marks : 200
Online Prelims TEST - 11 (SUBJECT WISE)
( InsightsIAS Mock Test Series for UPSC Preliminary Exam 2020 ) Mark Scored : 53.33

Your Answer :
Correct Answer : C

Answer Justification :

Management Effectiveness Evaluation (MEE) report is released by National Tiger Conservation


Authority (NTCA).

It ranks tiger reserves on numerous parameters.

https://www.thehindu.com/news/national/karnataka/karnataka-second-in-tiger-numbers-but-data-ver
y-robust-experts/article28751036.ece

88 AMLD-5, sometime seen in the news, is related to

A. Money Laundering 4
B. Regulation of Press 1 03
C. Volte Technology
4 64
5
D. None of the above
- 87
Your Answer :
.com
Correct Answer : A a i l
g m
2@
m2
Answer Justification :
a
n up
Fifth EU Anti-Money Laundering Directive (AMLD 5) was published in the official journal of the
a
European Union. The AMLD5smodifies the fourth Anti-Money Laundering Directive (AMLD4)
released only in 2015. - d
a
D as
The EU Commission proposed the revised AMLD in July 2016 as part of its Action Plan against
p am
terrorism announced in February 2016, after the attacks in Paris and Brussels, and as a reaction to
nu Papers published
A
the Panama in April 2016.

https://globalcompliancenews.com/eu-5th-anti-money-laundering-directive-published-20180716/

89 Consider the following statements regarding E-Vidhan


1. It is a Mission Mode Project (MMP) comes under the Digital India Programme.
2. Ministry of Parliamentary Affairs (MoPA) is the nodal ministry for its implementation in all the 31
States/UTs with Legislatures.

Which of the statements given above is/are correct?


A. 1 only
B. 2 only
C. Both 1 and 2
D. Neither 1 nor 2

Your Answer : C

prelims.insightsonindia.com 53
© Insights Active Learning | All rights reserved - 131815. You may not reproduce, distribute or exploit the contents in any form without
written permission by copyright owner. Copyright infringers may face civil and criminal liability
Total Marks : 200
Online Prelims TEST - 11 (SUBJECT WISE)
( InsightsIAS Mock Test Series for UPSC Preliminary Exam 2020 ) Mark Scored : 53.33

Correct Answer : C

Answer Justification :

E-Vidhan:
It is a Mission Mode Project (MMP) comes under the Digital India Programme. Ministry of
Parliamentary Affairs (MoPA) is the ‘Nodal Ministry’ for its implementation in all the 31 States/UTs
with Legislatures.

The funding for e-Vidhan is provided by the MoPA and technical support by Ministry of Electronics
and Information Technology (MietY). The funding of NeVA is through Central Sponsored Scheme
i.e. 60:40; and 90:10 for North East & hilly States and 100% for UTs.

https://pib.gov.in/newsite/PrintRelease.aspx?relid=186569

34
90 Consider the following statements
0
4in12005.
1. India is not a member of ASEAN regional forum.
4 6
2. India has been participating in the East Asia Summit since its very inception
3. Mekong Ganga Cooperation consists of six member countries 75
-8
m
Which of the statements given above is/are correct?
i l .co
ma
A. 2 only
B. 3 only g
C. 1 and 2 only 2 2@
D. 2 and 3 only am p
u
s an
Your Answer : D a
Correct Answer : D
s -d
Da
am
Answer Justification :
n up
A Regional Forum was launched in 1993. It is twenty-seven-member multilateral grouping
ASEAN
developed to facilitate cooperation on political and security issues to contribute to regional
confidence-building and
preventive diplomacy.

India is a member country of ASEAN Regional Forum

India has been participating in the East Asia Summit (EAS) since
its very inception in 2005. The EAS consists of 10 ASEAN nations (Indonesia,
Thailand, Singapore, Malaysia, the Philippines, Vietnam, Myanmar, Cambodia,
Brunei and Laos), Australia, China, India, Japan, New Zealand, South Korea, Russia and the U.S

Mekong Ganga Cooperation (MGC) was established in 2000. It comprises six


member countries namely India, Thailand, Myanmar, Cambodia, Laos and Vietnam.

91 Intermediate Range Nuclear Forces (INF) treaty, sometimes seen in the news, is treaty between

prelims.insightsonindia.com 54
© Insights Active Learning | All rights reserved - 131815. You may not reproduce, distribute or exploit the contents in any form without
written permission by copyright owner. Copyright infringers may face civil and criminal liability
Total Marks : 200
Online Prelims TEST - 11 (SUBJECT WISE)
( InsightsIAS Mock Test Series for UPSC Preliminary Exam 2020 ) Mark Scored : 53.33

A. United States and Iran


B. Russia and China
C. United States and China
D. Russia and United States

Your Answer : B
Correct Answer : D

Answer Justification :

Intermediate range Nuclear forces (INF) is a cold war era treaty between Russia and USA to end
arms race. US has recently announced that it will withdraw from the Intermediate-Range Nuclear
Forces (INF) Treaty.

https://www.thehindu.com/news/international/us-formally-withdraws-from-inf-arms-treaty/article288
00961.ece
0 34
1
4Summit
92 Consider the following statements regarding World Sustainable Development 4 6 (WSDS)
7 5
-8
1. It is a platform to accelerate action towards sustainable development and climate change.
2. It is an annual event organized by World Bank
. c om
Which of the statements given above is/are correct? ai
l
A. 1 only
@ gm
B. 2 only 2 2
m
C. Both 1 and 2
u pa
D. Neither 1 Nor 2 n
a sa
Your Answer : C s -d
Correct Answer : D A
a
p am
uJustification :
An
Answer

The World Sustainable Development Summit (WSDS) is The Energy and Resources Institute's
annual event. The theme of the 2020 edition of the Summit is 'Towards 2030 Goals: Making the
Decade Count'.

The World Sustainable Development Summit brings together Nobel laureates, political leaders,
decision-makers from bilateral and multilateral institutions, business leaders, high-level
functionaries from the diplomatic corps, scientists and researchers, media personnel, and members
of civil society; on a common platform to deliberate on issues related to sustainable development.

Post the adoption of the Sustainable Development Goals and the signing of the Paris Agreement, the
World Sustainable Development Summit aims to provide long-term solutions for the benefit of the
global community by assembling the various stakeholders on a single platform and initiating a step
in the direction of achieving constructive action in combating issues pertinent to the future of
humanity.

https://en.wikipedia.org/wiki/World_Sustainable_Development_Summit

prelims.insightsonindia.com 55
© Insights Active Learning | All rights reserved - 131815. You may not reproduce, distribute or exploit the contents in any form without
written permission by copyright owner. Copyright infringers may face civil and criminal liability
Total Marks : 200
Online Prelims TEST - 11 (SUBJECT WISE)
( InsightsIAS Mock Test Series for UPSC Preliminary Exam 2020 ) Mark Scored : 53.33

93 The largest reserves of thorium in India is located in

A. Gujarat
B. Kerala
C. Andhra Pradesh
D. Odisha

Your Answer : B
Correct Answer : C

Answer Justification :

Thorium Reserves

4
1 03
4 64
5
- 87
.com
a i l
g m
2 2@
am
up
s an
- da
s
Da
m
u pa
An
94 AG600, world’s largest amphibious aircraft indigenously designed and built

A. Japan
B. Russia
C. China
D. USA

Your Answer :
Correct Answer : C

Answer Justification :

AG600, world’s largest amphibious aircraft was indigenously designed and built by China. It carried
out its first take-off and landing tests.

prelims.insightsonindia.com 56
© Insights Active Learning | All rights reserved - 131815. You may not reproduce, distribute or exploit the contents in any form without
written permission by copyright owner. Copyright infringers may face civil and criminal liability
Total Marks : 200
Online Prelims TEST - 11 (SUBJECT WISE)
( InsightsIAS Mock Test Series for UPSC Preliminary Exam 2020 ) Mark Scored : 53.33

https://economictimes.indiatimes.com/news/international/world-news/china-built-worlds-largest-amp
hibious-plane/articleshow/66290512.cms?from=mdr

95 Freedom of Expression Report, often seen in news, is published

A. Amnesty International
B. Conservational International
C. Reporter’s without border
D. PEN International

Your Answer : A
Correct Answer : D

Answer Justification :

PEN International released its annual “Freedom of Expression Report” at the end3of4its 84th
0 in India in the
41
Congress held in Pune, says the climate for free expression has severely deteriorated
6
last few years. 4 5
7
https://pen-international.org/ -8
o m
a il.c
m
@g
96 Consider the following pairs
Index
2 2 Released by
m
pa
1. Human Capital Index OECD
2. Global Hunger Index n u FAO
a sa
- d
3. Global Competitiveness Index World Bank

D as
Which of the pairs given above is/are matched correctly?
p
A. 2 and a3 m
u only
B.A1nonly
C. 1 and 3 only
D. None

Your Answer : A
Correct Answer : D

Answer Justification :

The World Bank releases Human Capital Index (HCI) as part of the World Development Report.

The GHI is calculated annually, and its results appear in a report issued in October each year.
Created in 2006, the GHI was initially published by the International Food Policy Research Institute
(IFPRI) and Welthungerhilfe. In 2007, the Irish NGO Concern Worldwide also became a co-
publisher.

The Global Competitiveness Report (GCR) is a yearly report published by the World Economic

prelims.insightsonindia.com 57
© Insights Active Learning | All rights reserved - 131815. You may not reproduce, distribute or exploit the contents in any form without
written permission by copyright owner. Copyright infringers may face civil and criminal liability
Total Marks : 200
Online Prelims TEST - 11 (SUBJECT WISE)
( InsightsIAS Mock Test Series for UPSC Preliminary Exam 2020 ) Mark Scored : 53.33

Forum.

97 Which of the following is/are the tributaries of Godavari?


1. Manjira
2. Purna
3. Indravati
4. Pranahita

Select the correct answer using the code given below


A. 2 and 3 only
B. 1, 2, 3 and 4
C. 2 , 3 and 4 only
D. 1, 3 and 4 only

Your Answer : D
4
Correct Answer : B
1 03
4 64
Answer Justification : 5
- 87
The Godavari is India's second longest river after the Ganga. mIts source is in Triambakeshwar,
c o
Maharashtra. It flows east for 1,465 kilometres (910 mi),l.draining the states of Maharashtra
(48.6%), Telangana (18.8%), Andhra Pradesh (4.5%), a
i
Chhattisgarh (10.9%), Odisha
m
@gthrough its extensive network of tributaries.
(5.7%),ultimately emptying into the Bay of Bengal
2
m 2
u pa
a n
s
- da
s
Da
m
u pa
An
98 Consider the following statements regarding Genome India Initiative
1. It is an initiative of Department of Biotechology (DBT) to scan nearly 20,000 Indian genomes over
the next five years.
2. The data generated would be accessible to researchers anywhere for analysis.

Which of the statements given above is/are correct?


A. 1 only
B. 2 only
C. Both 1 and 2
D. Neither 1 nor 2

Your Answer : C
Correct Answer : C

Answer Justification :

prelims.insightsonindia.com 58
© Insights Active Learning | All rights reserved - 131815. You may not reproduce, distribute or exploit the contents in any form without
written permission by copyright owner. Copyright infringers may face civil and criminal liability
Total Marks : 200
Online Prelims TEST - 11 (SUBJECT WISE)
( InsightsIAS Mock Test Series for UPSC Preliminary Exam 2020 ) Mark Scored : 53.33

Genome India Initiative

It is an initiative of Department of Biotechology (DBT) to scan nearly 20,000 Indian genomes over
the next five years in a two-phase exercise, and develop diagnostic tests that can be used to test for
cancer.

Significance:

1. The data generated would be accessible to researchers anywhere for analysis. This would be
through a proposed National Biological Data Centre envisaged in a policy called the
‘Biological Data Storage, Access and Sharing Policy’, which is still in early stages of
discussion.

2. As the genetic landscape differs across the world, it is necessary that genetic data is shared in
order to derive greater knowledge from information and serve the purpose of enabling better
4
treatment outcomes. 03
6 41
5 4
87 and genetic variations for
3. The GenomeIndia initiative will pave the way for identifying genes
-
common diseases, treating Mendelian disorders, enabling
o m the transformation of the Precision
c
il. healthcare of the general population in
Medicine landscape in India, and thus improving the
our country. m a
@ g
22
am
https://www.thehindu.com/sci-tech/science/biotechnology-department-will-scan-20000-indian-genom
p
es/article28815520.ece n u
s a
- da
s
99 Bathukamma Festival,asometimes seen in the news, is celebrated in
D
am
up Nadu
A. Tamil
n
B.ATelangana
C. Karnataka
D. Odisha

Your Answer : A
Correct Answer : B

Answer Justification :

Bathukamma:

The festival was traditionally celebrated by women across the state.

Every year this festival is celebrated as per Telugu version of Hindu calendar in
the Bhadrapada Amavasya, also known as Mahalaya Amavasya, usually in

prelims.insightsonindia.com 59
© Insights Active Learning | All rights reserved - 131815. You may not reproduce, distribute or exploit the contents in any form without
written permission by copyright owner. Copyright infringers may face civil and criminal liability
Total Marks : 200
Online Prelims TEST - 11 (SUBJECT WISE)
( InsightsIAS Mock Test Series for UPSC Preliminary Exam 2020 ) Mark Scored : 53.33

September–October of Gregorian calendar.

In Telugu, ‘Bathukamma’ means ‘Mother Goddess come Alive’.

It is the state festival of Telangana.

https://www.insightsonindia.com/2019/09/30/insights-daily-current-affairs-pib-30-september-2019/

100 Consider the following statements regarding Saraswati Samman


1. It is the annual award given to an outstanding literary work in any Indian language mentioned in
Schedule VIII of the Constitution.
2. The award is presented by National Academy of Letters.

4
Which of the statements given above is/are correct?
1 03
A. 1 only
4 64
B. 2 only 5
C. Both 1 and 2
- 87
D. Neither 1 nor 2
.com
a i l
Your Answer :
g m
Correct Answer : A
2 2@
am
Answer Justification :
up
s an
Saraswati Samman: da
a s-
D Samman is the annual award given to an outstanding literary work in any
Eligibility: Saraswati
m
a mentioned in Schedule VIII of the Constitution and published in 10 years preceding
Indian language
u p
An
the specified award year.

Significance: It is the highest recognition in the field of Indian literature in the country and carries
a citation, a plaque and award money of ₹15 lakh.

The award is presented by the KK Birla Foundation, a literary and cultural organisation that
also gives the Vyas Samman for Hindi, and Bihari Puraskar for Hindi and Rajasthani writers of
Rajasthan.

https://www.insightsonindia.com/2019/09/30/insights-daily-current-affairs-pib-30-september-2019/

prelims.insightsonindia.com 60
© Insights Active Learning | All rights reserved - 131815. You may not reproduce, distribute or exploit the contents in any form without
written permission by copyright owner. Copyright infringers may face civil and criminal liability

You might also like